You are on page 1of 86

Total Marks : 200

Online Prelims TEST - 31 (TEXTBOOK)


( InsightsIAS Mock Test Series for UPSC Preliminary Exam 2020 ) Mark Scored : 0

1 Consider the following statements about certain constitutional provisions.


1. Provisions relating to persons with disability and old age are provided for both in Directive
Principles of State Policy (Part V) and Fundamental Duties (Part IVA).
2. Provisions relating to prevention of substance abuse are provided for in Fundamental Rights (Part
III).

Select the correct answer using the codes below.


A. 1 only
B. 2 only
C. Both 1 and 2
D. None of the above

Correct Answer : D

Answer Justification :

Justification: The only provisions relating to persons with disability and old age:

A41. Right to work, to education and to public assistance in certain cases: (Directive Principles of
State Policy) - The State shall, within the limits of its economic capacity and development, make
effective provision for securing the right to work, to education and to public assistance in cases of
unemployment, old age, sickness and disablement, and in other cases of undeserved want.

The only provisions relating to prevention of substance abuse are in IV. Directive Principles of State
Policy

A47. Duty of the State to raise the level of nutrition and the standard of living and to improve public
health – The State shall regard the raising of the level of nutrition and the standard of living of its
people and the improvement of public health as among its primary duties and, in particular, the
State shall endeavour to bring about prohibition of the consumption except for medicinal
purposes of intoxicating drinks and of drugs which are injurious to health.

Q Source: http://socialjustice.nic.in/writereaddata/UploadFile/IV.pdf

http://socialjustice.nic.in/writereaddata/UploadFile/V.pdf

2 Consider the following with regards to Uranium contamination of groundwater in India.


1. Uranium is a naturally occurring element in ocean water.
2. Indian Standard IS 10500: 2012 for Drinking Water specification has not specified the maximum
acceptable limits for Uranium in drinking water.
3. A recent report tabled in Parliament states that the North-eastern region of India is witnessing the
most Uranium contamination in India.

Select the correct answer using the codes below.


A. 1 and 2 only
B. 3 only
C. 1 and 3 only

1
Total Marks : 200
Online Prelims TEST - 31 (TEXTBOOK)
( InsightsIAS Mock Test Series for UPSC Preliminary Exam 2020 ) Mark Scored : 0

D. 1 only

Correct Answer : D

Answer Justification :

Justification: Statement 1: Uranium is weakly radioactive and remains so because of its long
physical half-life (4.468 billion years for uranium-238).

Uranium is a naturally occurring metal, which is widespread in nature. It is present in the


ocean and certain types of soils and rocks, especially granite.
The biological half-life (the average time it takes for the human body to eliminate half the
amount in the body) for uranium is about 15 days.
This is the highest-numbered element to be found naturally in significant quantities on earth.
It is considered to be more plentiful than antimony, beryllium, cadmium, gold, mercury, silver,
or tungsten.
It is about as abundant as tin, arsenic or molybdenum.

Statement 2: A report on Uranium Contamination in ground water in Parliament was tabled


recently.

The Indian Standard IS 10500: 2012 for Drinking Water specification has specified the maximum
acceptable limits for radioactive residues as alpha and beta emitters, values in excess of which
render the water not suitable.

These requirements take into account all radioactive elements including uranium. No individual
radioactive elements have been specifically identified.

As per Bureau of Indian Standard (BIS), maximum permissible limit of Uranium is 0.03 mg/l (as per
WHO provisional guidelines) in all drinking water standards after following due process.

Statement 3: Affected states:

A report brought out by Duke University, USA in association with Central Ground Water Board and
State Ground Water departments states that Andhra Pradesh, Chhattisgarh, Gujarat, Haryana,
Himachal Pradesh, Jharkhand, Karnataka, Maharashtra, Odisha, Punjab, Rajasthan, Tamil Nadu,
Telangana, Uttar Pradesh, West Bengal and Jammu & Kashmir have localised occurrence of
Uranium concentration.

Learning: Main factors responsible for uranium contamination:

Amount of uranium contained in an aquifer’s rocks.


Water-rock interactions that cause the uranium to be extracted from those rocks.
Oxidation conditions that enhance the extracted uranium’s solubility in water.
The interaction of the extracted uranium with other chemicals in the groundwater, such as
bicarbonate, which can further enhance its solubility.
Human factors such as groundwater-table decline and nitrate pollution may be exacerbating
the problem.

2
Total Marks : 200
Online Prelims TEST - 31 (TEXTBOOK)
( InsightsIAS Mock Test Series for UPSC Preliminary Exam 2020 ) Mark Scored : 0

Q Source:
https://www.insightsonindia.com/2020/03/17/insights-daily-current-affairs-pib-summary-17-march-2
020/

3 Innate immunity employs which of the following barriers in humans?


1. Acid in the stomach
2. Eye tears
3. Mucus coating lining the respiratory tract

Select the correct answer using the codes below.


A. 1, 2 and 3
B. 1 and 2 only
C. 1 and 3 only
D. 2 only

Correct Answer : A

Answer Justification :

Learning: Innate immunity is non-specific type of defence, which is present at the time of birth.
This is accomplished by providing different types of barriers to the entry of the foreign agents into
our body.

Innate immunity consists of four types of barriers.

These are —

Physical barriers: Skin on our body is the main barrier which prevents entry of the micro-
organisms. Mucus coating of the epithelium lining the respiratory, gastrointestinal and
urogenital tracts also help in trapping microbes entering our body.
Physiological barriers: Acid in the stomach, saliva in the mouth, tears from eyes–all prevent
microbial growth.
Cellular barriers: Certain types of leukocytes (WBC) of our body prevent microbial growth.

Q Source: 12th NCERT Biology

4 Consider the following matches with regards to certain provisions of the


Indian Penal Code (IPC) see in news repeatedly.
1. Section 188: A. Unlawful assembly
2. Sections 269 & B. Malignant act that is likely to spread the
270: infection of any disease dangerous to life
C. Disobedience to order duly promulgated by
3. Section 141:
public

Select the correct answer using the codes below.


A. 1A, 2B, 3C

3
Total Marks : 200
Online Prelims TEST - 31 (TEXTBOOK)
( InsightsIAS Mock Test Series for UPSC Preliminary Exam 2020 ) Mark Scored : 0

B. 1C, 2B, 3A
C. 1B, 2A, 3C
D. 1C, 2A, 3B

Correct Answer : B

Answer Justification :

Justification: Statement 1: The Epidemic Diseases Act, 1897 lays down punishment as per Section
188 of the Indian Penal Code, 1860, for flouting orders issued by various state governments to
contain the spread of COVID-19.

In the past, the Act has been routinely enforced across the country for dealing with outbreaks of
diseases such as swine flu, dengue, and cholera. Its penal provisions are currently being invoked by
states to contain the COVID-19 pandemic.

Section 188 relates to Disobedience to order duly promulgated by public servant.


It says violators can be punished with simple imprisonment for a term which may extend to
one month or with fine which may extend to two hundred rupees, or with both;
and if such disobedience causes or tends to cause danger to human life, health or safety, or
causes or tends to cause a riot or affray, shall be punished with imprisonment of either
description for a term which may extend to six months, or with fine which may extend to one
thousand rupees, or with both.

Statement 2: Sections 269 & 270 IPC invoked are being invoked against persons who malignantly
do any act which is likely to spread the infection of any disease dangerous to life.

Sections 269 (negligent act likely to spread infection of disease dangerous to life) and 270
(malignant act likely to spread infection of disease dangerous to life) come under Chapter XIV
of the IPC.
While Section 269 provides for a jail term of six months and/or fine, Section 270 provides for a
jail term of two years and/or fine.
In Section 270, the word ‘malignantly’ indicates a deliberate intention on the part of the
accused.

Statement 3: The definition of 'unlawful assembly', according to Indian law, is laid down in Section
141 of the Indian Penal Code. According to this section, an assembly of five or more persons
becomes unlawful when its purpose is or becomes:

To overawe by criminal force, or show of criminal force, the Central or any State Government
or Parliament or the Legislature of any State, or any public servant in the exercise of the
lawful power of such public servant;
To resist the execution of any law, or of any legal process;
To commit any mischief or criminal trespass, or other offence etc;

Q Source: In news

4
Total Marks : 200
Online Prelims TEST - 31 (TEXTBOOK)
( InsightsIAS Mock Test Series for UPSC Preliminary Exam 2020 ) Mark Scored : 0

5 Consider the following statements.


1. The Saka era was founded by King Gandharvasena of the Shatavahana dynasty.
2. The Saka era is used to indicate the year by Gazette of India and All India Radio News Broadcast.

Which of the above is/are correct?


A. 1 only
B. 2 only
C. Both 1 and 2
D. None of the above

Correct Answer : B

Answer Justification :

Learning: It is used by: Gazette of India, All India Radio News Broadcast, and Government of India.

The Saka era is believed to have been founded by King Shalivahana of the Shatavahana
dynasty.
Works like the Muhurta-Martanda suggests that the beginning of Saka Era is calculated from
the birth of Shalivahana, while the Kalpa Pradipa written in 1300 CE suggests that it marks
the victory of Shalivahana over Vikramaditya.
The use of this calendar era survived into the Gupta period and became part of Hindu
tradition following the decline of Buddhism in India.
It was in widespread use by the 6th to 7th centuries, e.g. in the works of Varahamihira and
Brahmagupta, and by the 7th century also appears in epigraphy in Hindu Southeast Asia.

Q Source: National Symbols

6 Members of the PM- CARES fund include


1. Union Home Minister
2. Union Defence Minister
3. Union Finance Minister
4. Cabinet Secretary
5. Union Minister for Housing and Urban Poverty Alleviation

Consider the following with regards to Select the correct answer using the codes below.
A. 1, 2 and 3 only
B. 1, 3 and 4 only
C. 1, 2, 3, 4 and 5
D. 2, 4 and 5 only

Correct Answer : A

Answer Justification :

5
Total Marks : 200
Online Prelims TEST - 31 (TEXTBOOK)
( InsightsIAS Mock Test Series for UPSC Preliminary Exam 2020 ) Mark Scored : 0

Justification: PM announced the PM-CARES fund to fight coronavirus outbreak.

PM- CARES fund stands for Prime Minister’s Citizen Assistance and Relief in Emergency Situations
Fund.

The prime minister is the chairman of the new public charitable trust, its members include the
defence minister, home minister and the finance minister.

It is a public charitable trust.

Contributions by corporates to this fund will be considered as social welfare spending under the
companies law.

It will be an emergency situation fund. This Fund will also cater to similar distressing situations, if
they occur in the times ahead.

Q Source:
https://www.insightsonindia.com/2020/03/30/insights-daily-current-affairs-pib-summary-30-march-2
020/

7 These species included in Appendix I of UN Convention on Migratory Species were recently in news:
1. Great Indian Bustard
2. Asiatic Cheetah
3. Bengal Florican

Select the correct answer using the codes below.


A. 2 only
B. 1 and 3 only
C. 1 and 2 only
D. 3 only

Correct Answer : B

Answer Justification :

Justification: S2: In the 2015 update of the IUCN Red List, the Asiatic cheetah is considered
regionally extinct in Iraq, Central Asia, Afghanistan, Pakistan and India.

S1 and S3: The Great Indian Bustard, Asian Elephant and Bengal Florican have been included in
Appendix I of UN Convention on Migratory Species. This was done at the ongoing 13th Conference
of the Parties (COP) to the Convention on Migratory Species (CMS) in Gandhinagar (Gujarat).

About Asian Elephant:

Found in India, Nepal, Bangladesh, Bhutan ans Myanmar. IUCN status: Endangered. It is also listed
in Appendix I of the Convention on International Trade in Endangered Species of Wild Fauna and
Flora (CITES) and Schedule I of the Wildlife (Protection) Act, 1972.

6
Total Marks : 200
Online Prelims TEST - 31 (TEXTBOOK)
( InsightsIAS Mock Test Series for UPSC Preliminary Exam 2020 ) Mark Scored : 0

About Great Indian Bustard:

It is one of the heaviest flying birds in the world. Its largest populations are found in the Indian
state of Rajasthan. State bird of Rajasthan. It is listed as ‘Critically Endangered’ on the IUCN Red
List. It is also listed in Appendix I of CITES and Schedule I of the Indian Wildlife (Protection) Act,
1972.

About Bengal Florican:

In India it is found in Uttar Pradesh, Assam and Arunachal Pradesh. It has been listed as ‘Critically
Endangered’ on the IUCN Red List. The bird is listed under Schedule I of the Wildlife Protection Act
of India, 1972 and Appendix I of CITES.

Q Source:
https://www.insightsonindia.com/2020/02/22/insights-daily-current-affairs-pib-summary-21-february
-2020/

8 Consider the following matches of rivers with their origin locations.


1. Brahmani : Mahabaleshwar
2. Mahi: Amarkantak
3. Pennar: Kolar
4. Godavari: Nashik

Which of the above is/are correct matches?


A. 1 and 2 only
B. 3 and 4 only
C. 1 and 4 only
D. 2 and 3 only

Correct Answer : B

Answer Justification :

Justification: The image below is a comprehensive source of information.

7
Total Marks : 200
Online Prelims TEST - 31 (TEXTBOOK)
( InsightsIAS Mock Test Series for UPSC Preliminary Exam 2020 ) Mark Scored : 0

Q Source: Additional Research: 9th NCERT Geography

9 The Ministry of Social Justice and Empowerment currently extends assistance to Non-Governmental
Organizations working for the welfare of disadvantaged sections of the society. Which of the following
types of organizations is/are eligible to get assistance?
1. Body registered under the Society's Registration Act, 1860.
2. Indian Red Cross Society or its branches
3. A for-profit body with at least two years of experience in social welfare

Select the correct answer using the codes below.


A. 1, 2 and 3
B. 2 only
C. 1 and 3 only
D. 1 and 2 only

Correct Answer : D

Answer Justification :

Justification: The ministry currently extends assistance to Non-Governmental Organizations


working for the welfare of disadvantaged sections of the society like Scheduled Castes, Other
Backward Classes, Minorities, the Aged, Drug Addicts, Street Children, Disabled under schemes
8
Total Marks : 200
Online Prelims TEST - 31 (TEXTBOOK)
( InsightsIAS Mock Test Series for UPSC Preliminary Exam 2020 ) Mark Scored : 0

separately designed to address these target groups.

Generally all these schemes support projects in the area of education, training, rehabilitation of the
targeted groups. Recurring expenditure on components such as honorarium to staff, other recurring
non-honorarium items such as rent, food expenses, contingencies, stipend to trainees/students,
transport allowances and non recurring items such as furniture, equipments, construction of
building etc are supported under these projects.

Who are eligible to apply for assistance?

Body registered under the Society's Registration Act, 1860.


Public Registered Trust.
Charitable Company licensed under section 25 of the Companies Act, 1958.
Indian Red Cross Society or its branches.
Other public institutions having a legal status.
Body should have been registered for at least two years at the time of applying for assistance,
and should not be running for profit to any individual or body of individuals.

What basic conditions are required to be fulfilled by an eligible organization?

Ans: The basic conditions are required to be fulfilled by an eligible organization are:

Should have been registered for two years


A minimum of two years experience on the field of activity would normally be insisted on
Financial soundness and capacity to bear at least 10% of the budgeted expenditure.

Can an NGO get assistance for various projects under different schemes?

Yes, provided the eligibility criterion and basic conditions are fulfilled. However, as a matter of
policy, the Ministry seeks to broad base the voluntary action movement and at the same time ensure
that there is wider sectoral as well as geographical coverage, by rationalizing its intervention on a
circumstantial and regional needs basis.

Q Source: http://socialjustice.nic.in/UserView/index?mid=74591

10 Consider the following with regards to


1. Capital Adequacy Ratio (CAR) is the ratio of a bank’s demand deposits to its total liability.
2. The enforcement of regulated levels of CAR is intended to protect depositors and promote stability
and efficiency of financial systems.
3. As per RBI norms, Indian scheduled commercial banks are required to maintain the CAR percentage
as stated in the Basel III Banking report.

Select the correct answer using the codes below.


A. 1 and 2 only
B. 2 only
C. 2 and 3 only
D. 1 and 3 only

9
Total Marks : 200
Online Prelims TEST - 31 (TEXTBOOK)
( InsightsIAS Mock Test Series for UPSC Preliminary Exam 2020 ) Mark Scored : 0

Correct Answer : B

Answer Justification :

Justification: What is Capital to Risk Weighted Assets Ratio (CRAR)?

The CRAR, also known as the Capital Adequacy Ratio (CAR), is the ratio of a bank’s capital to
its risk. It is a measure of the amount of a bank’s core capital expressed as a percentage of its
risk-weighted asset.
It is decided by central banks and bank regulators to prevent commercial banks from taking
excess leverage and becoming insolvent in the process.

Statement 1: All demand deposit is bank’s liability. Moreover, not all liabilities are classified as risky
assets. So, S1 is an incorrect statement.

Why CRAR was enforced?

The enforcement of regulated levels of this ratio is intended to protect depositors and
promote stability and efficiency of financial systems around the world. It determines the
bank’s capacity to meet the time liabilities and other risks such as credit risk, operational risk,
etc.

Statement 3: The Basel III norms stipulated a capital to risk weighted assets of 8%.

However, as per RBI norms, Indian scheduled commercial banks are required to maintain a CRAR
of 9%.

Learning: The Cabinet Committee on Economic Affairs has given its approval for continuation of
the process of recapitalization of Regional Rural Banks (RRBs) by providing minimum regulatory
capital to RRBs for another year beyond 2019-20, that is, up to 2020-21.

The recapitalisation process of RRBs was approved by the cabinet in 2011 based on the
recommendations of a committee set up under the Chairmanship of K C Chakrabarty.

The National Bank for Agriculture and Rural Development (NABARD) identifies those RRBs, which
require recapitalisation assistance to maintain the mandatory CRAR of 9% based on the CRAR
position of RRBs, as on 31st March of every year.

This is for those RRBs which are unable to maintain minimum Capital to Risk weighted Assets Ratio
(CRAR) of 9%, as per the regulatory norms prescribed by the Reserve Bank of India.

Q Source: https://www.insightsonindia.com/2020/03/26/145240/

11 Consider the following statements.


1. Yakshagana is a traditional theatre form of Kerala.
2. Yakshagana is a temple art form that depicts mythological stories and Puranas.

Select the correct answer using the codes below.


A. 1 only

10
Total Marks : 200
Online Prelims TEST - 31 (TEXTBOOK)
( InsightsIAS Mock Test Series for UPSC Preliminary Exam 2020 ) Mark Scored : 0

B. 2 only
C. Both 1 and 2
D. None of the above

Correct Answer : B

Answer Justification :

Justification: Yakshagana is a traditional theatre form of Karnataka.

It is a temple art form that depicts mythological stories and Puranas.

It is performed with massive headgears, elaborate facial make-up and vibrant costumes and
ornaments.

Usually recited in Kannada, it is also performed in Malayalam as well as Tulu (the dialect of south
Karnataka).

It is performed with percussion instruments like chenda, maddalam, jagatta or chengila (cymbals)
and chakratala or elathalam (small cymbals).

More than 900 Yakshagana scripts, including the ones printed in 1905 and 1907, have now been
digitised and made available online for free, thanks to the voluntary community effort by some
Yakshagana lovers who did it under the banner Yakshavahini, a registered trust.

Q Source:
https://www.insightsonindia.com/2020/03/24/insights-daily-current-affairs-pib-summary-24-march-2
020/

12 Consider the following with regards to Technology development related institutions in India.
1. Technology Development Board (TDB) is a statutory body that primarily aims to eliminate the
import of technology in the near future and promotes development of indigenous technology.
2. TDB provides equity capital or loans to industrial concerns and financial assistance to research and
development institutions.
3. Global Innovation & Technology Alliance (GITA) was established in order to stimulate private
sector’s investment in R&D.
4. GITA is a joint venture between World Bank (WB) and TDB.

Select the correct answer using the codes below.


A. 1, 2, 3 and 4
B. 2, 3 and 4 only
C. 2 and 3 only
D. 1 and 4 only

Correct Answer : C

11
Total Marks : 200
Online Prelims TEST - 31 (TEXTBOOK)
( InsightsIAS Mock Test Series for UPSC Preliminary Exam 2020 ) Mark Scored : 0

Answer Justification :

Justification: Statement 1 and 2: About Technology Development Board:

It is a statutory body established by Technology Development Board Act, 1995.

Objective: To promote development and commercialization of indigenous technology and adaptation


of imported technology for much wider application.

Roles and functions of the board:

Encourage enterprises to take up technology-oriented products.


Provide equity capital or loans to industrial concerns and financial assistance to research and
development institutions.

Statement 3 and 4: About GITA:

In order to stimulate private sector’s investment in R&D, TDB has established Global
Innovation & Technology Alliance (GITA).
It is a joint venture between Confederation of Indian Industry (CII) and TDB with an equity
contribution of 51:49 respectively.
The GITA will assist Department of Science & Technology (DST) in implementing industrial
research and development programme with different countries under bilateral and
multilateral science and technology cooperation agreements.

Q Source:
https://www.insightsonindia.com/2020/03/23/insights-daily-current-affairs-pib-summary-23-march-2
020/

13 Consider the following with regards to Special Central Assistance (SCA) to Scheduled Castes Sub-
Plan (SCSP) where 100% assistance is given as an additive to SCSP of the States/ UTs on the basis of
certain criteria. These criteria include
1. Extent of protected forest area in the state
2. Relative backwardness of the states/UT
3. Geographical area of the state

Select the correct answer using the codes below.


A. 2 only
B. 1, 2 and 3
C. 1 and 3 only
D. 1 only

Correct Answer : A

Answer Justification :

Justification: Special Central Assistance (SCA) to Scheduled Castes Sub-Plan (SCSP): It is a


policy initiative for development of Scheduled Castes in which 100 % assistance is given as an

12
Total Marks : 200
Online Prelims TEST - 31 (TEXTBOOK)
( InsightsIAS Mock Test Series for UPSC Preliminary Exam 2020 ) Mark Scored : 0

additive to SCSP of the States/ UTs on the basis of certain criteria such as SC population of the
States/UTs, relative backwardness of States/UTs, percentage of SC families in the States/ UTs
covered by composite economic development programmes in the State Plan to enable them to cross
the poverty line, etc.

It is an umbrella strategy to ensure flow of targeted financial and physical benefits from all the
general sectors of development for the benefit of Scheduled Castes. Under this Scheme, the States
/UTs are required to formulate and implement Special Component Plan (SCP) for Scheduled Castes
as part of their annual plans by earmarking resources.

Learning: Scheme of Assistance to Scheduled Castes Development Corporations


(SCDCs): Share Capital contribution is released to the State Scheduled Castes Development
Corporations (SCDCs) under a Centrally Sponsored Scheme in the ratio of 49:51 between Central
Government and State Governments.

There are in total 27 such State-level Corporations which are working for the economic
development of Scheduled Castes, although some of these Corporations are also catering to the
requirements of other weaker sections of the Society, e.g. Scheduled Tribes, OBCs, Minorities etc.

The main functions of SCDCs include identification of eligible SC families and motivating them to
undertake economic development schemes, sponsoring the schemes to financial institutions for
credit support, providing financial assistance in the form of the margin money at a low rate of
interest, providing subsidy out of the funds made available to the States under the Scheme of
Special Central Assistance to Scheduled Castes Sub Plan of the States to reduce the repayment
liability and providing necessary tie up with other poverty alleviation programmes.

The SCDCs are playing an important role in providing credit and missing inputs by way of margin
money loans and subsidy to the target group. The SCDCs finance the employment oriented schemes
covering diverse areas of economic activities which inter-alia include (i) agriculture and allied
activities including minor irrigation (ii) small scale industry (iii) transport and (iv) trade and service
sector.

Q Source: http://socialjustice.nic.in/SchemeList/Send/38?mid=24541

http://socialjustice.nic.in/UserView/index?mid=19536

14 Which of the following is NOT associated with the Paleozoic period?

A. Formation of extensive forests


B. Appearance of seed-bearing plants
C. Absence of limbed vertebrates
D. Increase in the diversity of the species of fishes

Correct Answer : C

Answer Justification :

13
Total Marks : 200
Online Prelims TEST - 31 (TEXTBOOK)
( InsightsIAS Mock Test Series for UPSC Preliminary Exam 2020 ) Mark Scored : 0

Justification: It is a geologic period and system of the Paleozoic, spanning 60 million years from
the end of the Silurian, 419.2 million years ago, to the beginning of the Carboniferous, 358.9 Mya.

Key features:

It is named after Devon, England, where rocks from this period were first studied.
The first significant adaptive radiation of life on dry land occurred during the Devonian.
Option A: Free-sporing vascular plants began to spread across dry land, forming extensive
forests which covered the continents.
Option B: By the end of the period the first seed-bearing plants appeared.
Option C: The ancestors of all four-limbed vertebrates (tetrapods) began adapting to walking
on land, as their strong pectoral and pelvic fins gradually evolved into legs.
Option D: Fish reached substantial diversity during this time, leading the Devonian to often be
dubbed the Age of Fishes.
Various terrestrial arthropods also became well-established.
The first ammonites, species of molluscs, appeared.
The palaeogeography was dominated by the supercontinent of Gondwana to the south, the
continent of Siberia to the north, and the early formation of the small continent of Euramerica
in between.

Q Source:
https://www.insightsonindia.com/2020/03/23/insights-daily-current-affairs-pib-summary-23-march-2
020/

15 Consider the following with regards to Base Erosion and Profit Shifting (BEPS).
1. BEPS refers to the phenomenon where companies invest in a host country via shell companies
situated in tax havens to reduce their tax liability.
2. The Multilateral Convention to Implement Tax Treaty Related Measures to Prevent BEPS is an
outcome of the World Economic Forum BEPS Project.
3. India recently ratified the Multilateral Convention to Implement Tax Treaty Related Measures to
Prevent BEPS.

Select the correct answer using the codes below.


A. 1 and 2 only
B. 2 and 3 only
C. 1, 2 and 3
D. 3 only

Correct Answer : D

Answer Justification :

Justification: Statement 1: Base erosion and profit shifting refers to the phenomenon where
companies shift their profits to other tax jurisdictions, which usually have lower rates, thereby
eroding the tax base in India. The companies need not invest via shell companies to be accused of
BEPS.

14
Total Marks : 200
Online Prelims TEST - 31 (TEXTBOOK)
( InsightsIAS Mock Test Series for UPSC Preliminary Exam 2020 ) Mark Scored : 0

Statement 2 and 3: India in July 2019 ratified the international agreement to curb base erosion and
profits shifting (BEPS)– Multilateral Convention to Implement Tax Treaty Related Measures.

The Convention is an outcome of the OECD / G20 BEPS Project to tackle base erosion and profit
shifting through tax planning strategies that exploit gaps and mismatches in tax rules to artificially
shift profits to low or no-tax locations where there is little or no economic activity, resulting in little
or no overall corporate tax being paid.

Learning: Under BEPS Action 13, all large multinational enterprises (MNEs) are required to
prepare a country-by-country (CbC) report with aggregate data on the global allocation of income,
profit, taxes paid and economic activity among tax jurisdictions in which it operates. This CbC
report is shared with tax administrations in these jurisdictions, for use in high level transfer pricing
and BEPS risk assessments.

The Base Erosion and Profit Shifting (BEPS) Action 13 report (Transfer Pricing Documentation and
Country-by-Country Reporting) provides a template for multinational enterprises (MNEs) to report
annually and for each tax jurisdiction in which they do business the information set out therein. This
report is called the Country-by-Country (CbC) Report.

What CBC contains?

Aggregated country-by-country information relating to the global allocation of income, the


taxes paid, and certain other indicators of a multi-national company.
A list of all the constituent entities of the multi-national company operating in a particular
jurisdiction and the nature of the main business activity of each constituent entity.

Q Source:
https://www.insightsonindia.com/2020/03/20/insights-daily-current-affairs-pib-summary-20-march-2
020/

https://www.oecd.org/tax/beps/beps-actions/action13/

16 Central Ground Water Authority (CGWA) has been constituted under the

A. Environment (Protection) Act, 1986


B. Water (Prevention and Control of Pollution) Act, 1974
C. Public Liability Insurance Act, 1991
D. Hazardous waste Handling and management act, 1989

Correct Answer : A

Answer Justification :

Justification: Some bodies related to water management:

Central Ground Water Authority (CGWA) has been constituted under Section 3(3) of the
‘Environment (Protection) Act, 1986’ for the purpose of regulation and control of ground water
development and management in the Country. CGWA grants ‘No Objection Certificates’ (NOC) for
15
Total Marks : 200
Online Prelims TEST - 31 (TEXTBOOK)
( InsightsIAS Mock Test Series for UPSC Preliminary Exam 2020 ) Mark Scored : 0

ground water abstraction in 23 States/UTs.

National Water Informatics Centre has been established under National Hydrology Project.

National Aquifer Mapping and Management program (NAQUIM) is implemented by Central Ground
Water Board (CGWB). It envisages mapping of aquifers and development of Aquifer Management
Plans to facilitate sustainable management of Ground Water Resources.

Q Source: Insights current events

17 Which of the following Policies, Acts, or Programmes have a bearing on the welfare and maintenance
of Senior Citizens?
1. National Policy on Ageing, 2017 (NPOA)
2. Maintenance and Welfare of Parents and Senior Citizens Act
3. Central Sector Scheme of “Integrated Programme for Older Persons”(IPOP)

Select the correct answer using the codes below.


A. 1 and 2 only
B. 1, 2 and 3
C. 2 and 3 only
D. 1 and 3 only

Correct Answer : C

Answer Justification :

Background: Constant increase in life expectancy due to improvement in the health care facilities
over the years is one of the main reasons for rapid increase in proportion of population of Senior
Citizens.

As per Census 2011, the total population of Senior Citizens (people aged 60 years and above) is
10.38 crore, of which population of males and females are 5.11 crore and 5.27 crore respectively.

The share of senior citizens in the total population as per Census 2011 is 8.57%. As per the May
2006 Report of the “Technical Group on Population Projections” constituted by the National
Commission on Population published by the Office of the Registrar General of India this share is
expected to increase to 10.70 % in 2021 and to 12.40% in 2026.

Justification: S1: No such policy was released in 2017.

The following Policies, Act, and Programmes which aim at welfare and maintenance of Senior
Citizens, especially for indigent senior citizens, are being dealt with in the Ageing Division:-

1. National Policy on Older Persons (NPOP) –The existing National Policy on Older Persons
(NPOP) was announced in January 1999 to reaffirm the commitment to ensure the well-being
of the Older Persons. The Policy envisaged State support to ensure financial and food
security, health care, shelter and other needs of Older Persons, equitable share in
development, protection against abuse and exploitation, and availability of services to improve

16
Total Marks : 200
Online Prelims TEST - 31 (TEXTBOOK)
( InsightsIAS Mock Test Series for UPSC Preliminary Exam 2020 ) Mark Scored : 0

the quality of their lives. Keeping in view the changing demographic pattern, socio-economic
needs of the Senior Citizens, social value system and advancement in the field of science and
technology over the last decade, a new National Policy for Senior Citizens is under finalization
to replace the NPOP, 1999.
2. Maintenance and Welfare of Parents and Senior Citizens Act, 2007 (MWPSC Act) - The
Maintenance and Welfare of Parents and Senior Citizens (MWPSC) Act, 2007 was enacted in
December 2007 to ensure need based maintenance for Parents and Senior Citizens and their
welfare.
3. Central Sector Scheme of “Integrated Programme for Older Persons”(IPOP) - Under the
Scheme, financial assistance is provided to Government/ Non-Governmental
Organizations/Panchayati Raj Institutions/ local bodies etc. for running and maintenance of
various projects for the welfare of indigent Senior Citizens under the Scheme of IPOP.

Q Source: http://socialjustice.nic.in/UserView/index?mid=47564

18 Government fixes Minimum Support Prices (MSPs) of 23 mandated crops. These include
1. Oilseeds
2. Cotton
3. Raw Jute

Select the correct answer using the codes below.


A. 1 only
B. 2 only
C. 1, 2 and 3
D. 2 and 3 only

Correct Answer : C

Answer Justification :

Justification: As of now, CACP recommends MSPs of 23 commodities, which comprise 7 cereals


(paddy, wheat, maize, sorghum, pearl millet, barley and ragi), 5 pulses (gram, tur, moong, urad,
lentil), 7 oilseeds (groundnut, rapeseed-mustard, soyabean, seasmum, sunflower, safflower,
nigerseed), and 4 commercial crops (copra, sugarcane, cotton and raw jute).

CACP submits its recommendations to the government in the form of Price Policy Reports every
year, separately for five groups of commodities namely Kharif crops, Rabi crops, Sugarcane, Raw
Jute and Copra. Before preparing aforesaid five pricing policy reports, the Commission draws a
comprehensive questionnaire, and sends it to all the state governments and concerned National
organizations and Ministries to seek their views.

Q Source:
https://www.insightsonindia.com/2020/03/18/insights-daily-current-affairs-pib-summary-18-march-2
020/

19 With reference to the Centre for Indian Knowledge Systems (CIKS), consider the following.

17
Total Marks : 200
Online Prelims TEST - 31 (TEXTBOOK)
( InsightsIAS Mock Test Series for UPSC Preliminary Exam 2020 ) Mark Scored : 0

1. It looks into the root of ancient knowledge systems to gain a strong understanding of their workings
and rationale.
2. It awards Geographical Indication (GI) tags to Indian heritage products.
3. It is responsible for regulating biopiracy in India.

Select the correct answer using the codes below.


A. 1 only
B. 2 and 3 only
C. 3 only
D. 1, 2 and 3

Correct Answer : A

Answer Justification :

Justification: CIKS was registered as an independent trust in 1995. The activities of the centre are
supported through grants from government agencies, private agencies, donations from individuals
and the sale of educational and training material.

Statement 1: For e.g. CIKS is developing methods for organic vegetable cultivation using recipes
found in Vrkshayurveda, for e.g. treating with milk and ghee will help latex yielding verities give a
bumper harvest

The understanding and knowledge gained by CIKS is then leveraged to develop solutions that
are practical and feasible in today's context.
Its community seed bank project is aimed at identifying important traditional seed varieties
and orienting the agricultural community towards conserving and cultivating them.

Learning: CIKS initiatives in biodiversity conservation have helped farmers in Tamil Nadu conserve
over 180 indigenous seed varieties - hence protecting the dwindling genetic base and giving
farmers a way to beat stress situations.

Q Source: http://www.ciks.org/vrksh1.htm

20 Consider the following about ExoMars.


1. It is an initiative of SpaceX.
2. It aims to examine the change in Martian orbit patterns.

Consider the following with regards to Select the correct answer using the codes below.
A. 1 only
B. 2 only
C. Both 1 and 2
D. None of the above

Correct Answer : D

18
Total Marks : 200
Online Prelims TEST - 31 (TEXTBOOK)
( InsightsIAS Mock Test Series for UPSC Preliminary Exam 2020 ) Mark Scored : 0

Answer Justification :

Justification: About ExoMars:

It is a joint endeavour between ESA and the Russian space agency, Roscosmos.
The primary goal of the ExoMars programme is to address the question of whether life has
ever existed on Mars.
Launch of ExoMars rover delayed to 2022.

The European-built Rosalind Franklin rover, named for the famed British chemist and X-ray
crystallographer whose work contributed to DNA research, recently passed final pre-launch thermal
and vacuum tests at an Airbus facility in Toulouse, France.

Rosalind Franklin is the first European Mars rover.

Q Source:
https://www.insightsonindia.com/2020/03/16/insights-daily-current-affairs-pib-summary-16-march-2
020/

21 Consider the following with regards to Pradhan Mantri Adarsh Gram Yojana (PMAGY).
1. It is being implemented for integrated development of Scheduled Tribes (ST) majority villages and
districts.
2. The scheme is funded jointly by the Centre and the States.
3. Each village selected for the scheme is to be adopted by a Member of Parliament (MP).

Select the correct answer using the codes below.


A. 1 and 3 only
B. 2 only
C. 1 and 2 only
D. None of the above

Correct Answer : B

Answer Justification :

Justification: Pradhan Mantri Adarsh Gram Yojana (PMAGY): The Centrally Sponsored Pilot
Scheme ‘Pradhan Mantri Adarsh Gram Yojana’ (PMAGY) is being implemented for integrated
development of Scheduled Castes (SC) majority villages having SC Population concentration > 50%.
Initially the scheme was launched in 1000 villages in 5 States viz. Assam, Bihar, Himachal Pradesh,
Rajasthan and Tamil Nadu. The Scheme was further revised w.e.f. 22.01.2015 and extended to 1500
SC majority villages in Punjab, Madhya Pradesh, Andhra Pradesh, Karnataka, Uttar Pradesh,
Telangana, Haryana, Chhattisgarh, Jharkhand, Uttarakhand, West Bengal and Odisha. The principal
objective of the Scheme is integrated development of SC Majority Villages:

1. ​Primarily through convergent implementation of the relevant Central and State Schemes;
2. By providing these villages Central Assistance in form of gap-filling funds to the extent of
Rs.20.00 lakh per village, to be increased by another 5 lakh if State make a matching

19
Total Marks : 200
Online Prelims TEST - 31 (TEXTBOOK)
( InsightsIAS Mock Test Series for UPSC Preliminary Exam 2020 ) Mark Scored : 0

contribution.
3. By providing gap-filling component to take up activities which do not get covered under the
existing Central and State Government Schemes are to be taken up under the component of
‘gap filling’.

To ensure all round development of the selected villages, so that they can indeed become ‘Adarsh
Grams’, the said Scheme has been recently revised to capture the Gaps in critical socio-economic
‘Monitorable Indicators’ as part of various sectors/domains. These domains include water and
sanitation, education, health and nutrition, agricultural best practices etc. amongst others.

Q Source: http://www.pmagy.gov.in/aboutPMAGY

22 Consider the following with regards to the National Commission for Backward Classes (NCBC).
1. The conditions of service and tenure of office of the Chairperson and other Members is determined
by the President.
2. It was converted into a statutory body recently.
3. The constitution empowers the President to specify socially and educationally backward classes in
various states and union territories.

Select the correct answer using the codes below.


A. 1 only
B. 2 and 3 only
C. 1 and 3 only
D. 1, 2 and 3

Correct Answer : C

Answer Justification :

Justification: Statement 1: 102nd Constitution Amendment Act, 2018 provides constitutional


status to the National Commission for Backward Classes (NCBC).

It has the authority to examine complaints and welfare measures regarding socially and
educationally backward classes.

Previously NCBC was a statutory body under the Ministry of Social Justice and Empowerment.

Statement 2: The Commission consists of five members including a Chairperson, Vice-Chairperson


and three other Members appointed by the President by warrant under his hand and seal.

The conditions of service and tenure of office of the Chairperson, Vice-Chairperson and other
Members is determined by President.

Statement 3: Article 338B provides authority to NCBC to examine complaints and welfare
measures regarding socially and educationally backward classes.

Article 342 A empowers President to specify socially and educationally backward classes in various
states and union territories. He can do this in consultation with Governor of concerned State.
20
Total Marks : 200
Online Prelims TEST - 31 (TEXTBOOK)
( InsightsIAS Mock Test Series for UPSC Preliminary Exam 2020 ) Mark Scored : 0

However, law enacted by Parliament will be required if list of backward classes is to be amended.

Learning: The commission investigates and monitors all matters relating to the safeguards
provided for the socially and educationally backward classes under the Constitution or under any
other law to evaluate the working of such safeguards.

It participates and advises on the socio-economic development of the socially and educationally
backward classes and to evaluate the progress of their development under the Union and any State.

It presents to the President, annually and at such other times as the Commission may deem fit,
reports upon the working of those safeguards. The President laid such reports before each House of
Parliament.

Q Source: MoSJE website

23 Cabinet recently approved the Modified Electronics Manufacturing Clusters (EMC 2.0) Scheme. The
scheme aims to
1. Create Plug & Play infrastructure facility for attracting investment in the electronics sector.
2. Establish Common Facility Centre (CFC) where there are a significant number of Electronics
System Design & Manufacturing (ESDM) and upgrade the common technical infrastructure.

Select the correct answer using the codes below.


A. 1 only
B. 2 only
C. Both 1 and 2
D. None of the above

Correct Answer : C

Answer Justification :

Justification: The scheme provides for development of world class infrastructure along with
common facilities and amenities through Electronics Manufacturing Clusters (EMCs).

The Scheme would support setting up of both Electronics Manufacturing Clusters (EMCs) and
Common Facility Centers (CFCs).

Where can these clusters be setup?

An Electronics Manufacturing Cluster (EMC) would set up in geographical areas of certain


minimum extent, preferably contiguous, where the focus is on development of basic
infrastructure, amenities and other common facilities for the ESDM units.
For Common Facility Centre (CFC), there should be a significant number of existing ESDM
units located in the area and the focus is on upgrading common technical infrastructure and
providing common facilities for the ESDM units in such EMCs, Industrial
Areas/Parks/industrial corridors.

Benefits of the scheme:

21
Total Marks : 200
Online Prelims TEST - 31 (TEXTBOOK)
( InsightsIAS Mock Test Series for UPSC Preliminary Exam 2020 ) Mark Scored : 0

Availability of ready infrastructure and Plug & Play facility for attracting investment in
electronics sector.
New investment in electronics sector
Jobs created by the manufacturing units;
Revenue in the form of taxes paid by the manufacturing units

Q Source:
https://www.insightsonindia.com/2020/03/23/insights-daily-current-affairs-pib-summary-23-march-2
020/

24 The Madrid Plan (MIPAA) based on a United Nations General Assembly declaration in the beginning
of the 21st CE is a/an international Plan of Action on

A. Mendicancy
B. Ageing
C. Substance abuse
D. Ecological conservation

Correct Answer : B

Answer Justification :

Justification: The United Nations Second World Assembly on Ageing (Madrid, 8-12 April 2002)
unanimously adopted the Madrid Political Declaration and International Plan of Action on Ageing,
2002. WHO’s contributions to the Assembly included the submission of a policy framework,1 and
the formulation of regional action plans for implementing the International Plan, notably by the
United Nations Economic Commission for Europe, the United Nations Economic and Social
Commission for Asia and the Pacific, and the United Nations Economic Commission for Latin
America and the Caribbean. Reports on the content of the policy framework and the outcomes of
the Second World Assembly were submitted to the Fifty-fifth World Health Assembly.

The Madrid International Plan of Action on Ageing reflects a global consensus on the social
dimensions of ageing that has evolved during preceding decades through multilateral activity and
work conducted at the United Nations. Pioneering efforts to address the challenges of ageing
began in the 1940s, shortly after the inception of the United Nations.

Learning: The Vienna Plan was the first international instrument for action on development issues
of ageing. It identified three priority areas: (a) the sustainability of development in a world where
the population is increasing in age; (b) the maintenance of good health and well-being to an
advanced age; and (c) the establishment of an appropriate and supportive environment for all age
groups. The purpose of the Vienna Plan was to help Governments in formulating their policies on
ageing, by guiding national and international efforts and strengthening capacities of Governments
and civil society organizations to deal effectively with demographic ageing.

Q Source: http://socialjustice.nic.in/ViewData/Details?mid=1524&catID=34

https://www.un.org/esa/socdev/ageing/documents/publications/regional-dimensions-ageing.pdf

22
Total Marks : 200
Online Prelims TEST - 31 (TEXTBOOK)
( InsightsIAS Mock Test Series for UPSC Preliminary Exam 2020 ) Mark Scored : 0

25 Consider the following with regards to Limited Liability Partnership (LLP).


1. In a LLP, one partner is not responsible or liable for another partner’s misconduct or negligence.
2. LLP is considered a body corporate and a legal entity separate from its partners.

Select the correct answer using the codes below.


A. 1 only
B. 2 only
C. Both 1 and 2
D. None of the above

Correct Answer : C

Answer Justification :

Justification: A Limited Liability Partnership (LLP) is a partnership in which some or all partners
have limited liability. It therefore exhibits elements of partnerships and corporations.

In an LLP, one partner is not responsible or liable for another partner’s misconduct or negligence.

Salient features of an LLP:

An LLP is a body corporate and legal entity separate from its partners. It has perpetual
succession.
Being the separate legislation (i.e. LLP Act, 2008), the provisions of Indian Partnership Act,
1932 are not applicable to an LLP and it is regulated by the contractual agreement between
the partners.
Every Limited Liability Partnership shall use the words “Limited Liability Partnership” or its
acronym “LLP” as the last words of its name.

Learning: Ministry of Corporate Affairs introduces the “Companies Fresh Start Scheme, 2020”
and revised the “LLP Settlement Scheme, 2020” to provide relief to law abiding companies and
Limited Liability Partnerships (LLPs) in the wake of COVID 19.

What do these schemes entail?

These schemes incentivise compliance and reduce compliance burden during the unprecedented
public health situation caused by COVID-19.

The schemes provide a one-time waiver of additional filing fees for delayed filings by the companies
or LLPs with the Registrar of Companies during the currency of the Schemes, i.e. during the period
starting from 1stApril, 2020 and ending on 30th September, 2020.

They also significantly reduce the related financial burden on them, especially for those with long
standing defaults, thereby giving them an opportunity to make a “fresh start”.

Both the Schemes also contain provision for giving immunity from penal proceedings, including
against imposition of penalties for late submissions.

23
Total Marks : 200
Online Prelims TEST - 31 (TEXTBOOK)
( InsightsIAS Mock Test Series for UPSC Preliminary Exam 2020 ) Mark Scored : 0

They also provide additional time for filing appeals before the concerned Regional Directors against
imposition of penalties, if already imposed.

Q Source:
https://www.insightsonindia.com/2020/03/31/insights-daily-current-affairs-pib-summary-31-march-2
020

26 Which of the following public agencies is/are implementing Skill Development Programmes?
1. National Scheduled Castes Finance and Development Corporation( NSFDC)
2. National Safai Karmacharis Finance and Development Corporation (NSKFDC)
3. National Backward Classes Finance and Development Corporation (NBCFDC)

Consider the following with regards to Select the correct answer using the codes below.
A. 1 only
B. 1 and 3 only
C. 1, 2 and 3
D. 2 only

Correct Answer : C

Answer Justification :

Justification and Learning: India has adopted skill development as a national priority and
assigned yearly targets to the Ministries which are being monitored regularly. Department of Social
Justice and Empowerment is participating effectively in the implementation of the skill development
programme and enabling action at the grass root level with the help of the three Finance
Corporations which are promoting economic empowerment of the target groups.

These Corporations finance self employment oriented income generating schemes for their target
groups and are basically ‘not for profit’ Companies. They are also implementing the Skill
Development Programmes as per the targets assigned to them,

They are : i. National Scheduled Castes Finance and Development Corporation( NSFDC) ii. National
Safai Karmacharis Finance and Development Corporation (NSKFDC) iii. National Backward Classes
Finance and Development Corporation (NBCFDC)

The progress made by the Corporations in imparting skills to the target group can be tracked
through the links given under:

1. NSFDC - http://www.nsfdc.nic.in/uniquepage.asp?ID_PK=43

2. NSKFDC - http://nskfdc.nic.in/training.html

3. NBCFDC- http://www.nbcfdc.org.in/sc_tscheme.html

Q Source: http://socialjustice.nic.in/writereaddata/UploadFile/skilldevelopment.pdf

24
Total Marks : 200
Online Prelims TEST - 31 (TEXTBOOK)
( InsightsIAS Mock Test Series for UPSC Preliminary Exam 2020 ) Mark Scored : 0

27 The main objectives of OceanSat-2 are to study


1. Surface winds and ocean surface strata
2. Monitoring of phytoplankton blooms in the ocean
3. Study of atmospheric aerosols and suspended sediments in the ocean water

Consider the following with regards to Select the correct answer using the codes below.
A. 2 only
B. 1 and 3 only
C. 1 and 2 only
D. 1, 2 and 3

Correct Answer : D

Answer Justification :

Justification: Launched in 2009, it is designed to provide service continuity for operational users
of the Ocean Colour Monitor (OCM) instrument on Oceansat-1.

The main objectives of OceanSat-2 are to study surface winds and ocean surface strata, observation
of chlorophyll concentrations, monitoring of phytoplankton blooms, study of atmospheric aerosols
and suspended sediments in the water.

The Indian National Centre for Ocean Information Services (INCOIS) has reported that Oceansat
Satellite data are used to prepare the Potential Fishing Zone (PFZ) advisories on the potential rich
fishing areas and provide to the sea faring fishermen in all states.

How are these zones identified?

This methodology utilizes data on chlorophyll concentration (Chl) obtained from ISRO’s Oceansat-2
satellite and the sea surface temperature from National Oceanic Atmospheric Administration
(NOAA / USA satellites).

Q Source:
https://www.insightsonindia.com/2020/03/18/insights-daily-current-affairs-pib-summary-18-march-2
020/

28 Consider the following statements.


1. The word ‘beggar’ or ‘beggary’ is not mentioned in any of the lists of the Seventh Schedule of the
Constitution.
2. “Relief of the disabled and unemployable” is a Concurrent subject in the Seventh Schedule of the
Constitution.

Consider the following with regards to Select the correct answer using the codes below.
A. 1 only
B. 2 only
C. Both 1 and 2
D. None of the above

25
Total Marks : 200
Online Prelims TEST - 31 (TEXTBOOK)
( InsightsIAS Mock Test Series for UPSC Preliminary Exam 2020 ) Mark Scored : 0

Correct Answer : A

Answer Justification :

Justification: The word ‘beggar’ or ‘beggary’ is not mentioned in any of the lists of the
Constitution. However, as per entry-9 of the State List in the Seventh Schedule of the Constitution,
“Relief of the disabled and unemployable” is a State subject.As per entry-15 of the Concurrent List,
“Vagrancy” is a concurrent subject.

As per information available at present, 20 States and 2 UTs have either enacted their own Anti
Beggary Legislation or adopted the legislation enacted by other States. Despite the fact that many
States/UTs have enacted laws relating to beggary, however, the provisions of these legislations
differ across the States and their status of implementation, including the measures taken for
rehabilitation of beggars, are also not uniform.

The Ministry of SJE’s approach towards addressing the problem of beggary is rehabilitative rather
than punitive. Accordingly, the Ministry is in the process formulating a Model Legislation on
Destitution which could be suitably adopted/adapted by the States/UTs and also formulating a
Scheme for Protection, Care and Rehabilitation of Destitutes.

The MOSJE has developed an online web-portal e-utthaan.gov.in for monitoring of Allocation for
Welfare of Scheduled Castes. Similarly, Ministry of Tribal Affairs has developed the online web-
portal stcmis.gov.in for monitoring of Allocation for Welfare of Scheduled Tribes.

Q Source: http://socialjustice.nic.in/UserView/index?mid=47564

29 Socio-economic Caste Census (SECC), 2011, is a study of socio economic status of both rural and
urban households coordinated by the Department of Rural Development. SECC used explicit
‘inclusion’ and ‘exclusion’ criteria to rank households in the census. Which of these is/are such
inclusion criteria?
1. Possession of a Kisan Credit Card
2. Manual scavenger families
3. Primitive tribal groups

Select the correct answer using the codes below.


A. 2 only
B. 2 and 3 only
C. 1, 2 and 3
D. 1 and 3 only

Correct Answer : B

Answer Justification :

Background: SECC-2011 is a study of socio economic status of rural and urban households and
allows ranking of households based on predefined parameters. SECC 2011 has three census

26
Total Marks : 200
Online Prelims TEST - 31 (TEXTBOOK)
( InsightsIAS Mock Test Series for UPSC Preliminary Exam 2020 ) Mark Scored : 0

components which were conducted by three separate authorities but under the overall coordination
of Department of Rural Development in the Government of India.

Census in Rural Area has been conducted by the Department of Rural Development (DoRD). Census
in Urban areas is under the administrative jurisdiction of the Ministry of Housing and Urban
Poverty Alleviation (MoHUPA). Caste Census is under the administrative control of Ministry of
Home Affairs: Registrar General of India (RGI) and Census Commissioner of India.

Ministry of Rural Development commenced the Socio-Economic Caste Census-2011 on 29th June,
2011 through a comprehensive door to door enumeration across the country.

Justification: Total Excluded Households (based on fulfilling any of the 14 parameters of


exclusion -

i. Motorized 2/3/4 wheeler/fishing boat.

ii. Mechanized 3-4 wheeler agricultural equipment.

iii. Kisan credit card with credit limit of over Rs. 50,000/-.

iv. Household member government employee.

v. Households with non-agricultural enterprises registered with government.

vi. Any member of household earning more than Rs. 10,000 per month.

vii. Paying income tax.

viii. Paying professional tax.

ix. 3 or more rooms with pucca walls and roof.

x. Owns a refrigerator.

xi. Owns landline phone.

xii. Owns more than 2.5 acres of irrigated land with 1 irrigation equipment.

xiii. 5 acres or more of irrigated land for two or more crop season.

xiv. Owning at least 7.5 acres of land or more with at least one irrigation equipment.

Automatically included (based on fulfilling any of the 5 parameters of inclusion -

i. Households without shelter.

ii. Destitute, living on alms.

iii. Manual scavenger families.

iv. Primitive tribal groups.

27
Total Marks : 200
Online Prelims TEST - 31 (TEXTBOOK)
( InsightsIAS Mock Test Series for UPSC Preliminary Exam 2020 ) Mark Scored : 0

v. Legally released bonded labour.

Q Source: MoSJE

30 Petroleum & Explosives Safety Organization is a regulatory authority with autonomous status under
the

A. Ministry of Heavy Industries


B. Ministry of Commerce and Industry
C. Ministry of Petroleum and Natural Gas
D. Ministry of Shipping

Correct Answer : B

Answer Justification :

Justification: It is a department under Department for the Promotion of Industry and Internal
Trade under Ministry of Commerce and Industry.

It is a regulatory authority with autonomous status.

It was established during the British India in 1890s as Department of Explosives and later expanded
to various other activities.

As a statutory authority, PESO is entrusted with the responsibilities under the Explosives Act, 1884;
Petroleum Act, 1934; Inflammable Substances Act, 1952, Environment (Protection Act), 1986.

Q Source:
https://www.insightsonindia.com/2020/03/27/insights-daily-current-affairs-pib-summary-27-march-2
020/

31 National Highway 231 of India is most likely to be a

A. Branch of a main highway


B. Highway running in east-west direction
C. Highway running in North-south direction
D. A national highway that connects two states together

Correct Answer : A

Answer Justification :

Justification: Numbering of highways is based on a consistent system. Three digit numbered


highways are secondary routes or branches of a main highway. So, A is the answer. NH 231 passes
through Ambedkarnagar- Banda (UP).

28
Total Marks : 200
Online Prelims TEST - 31 (TEXTBOOK)
( InsightsIAS Mock Test Series for UPSC Preliminary Exam 2020 ) Mark Scored : 0

All North-South highways will carry EVEN number

All East-West highways will have ODD numbers


All major Highways will be single digit or double digit in number.
North-South highways will increase their numbers from East to West. For example, a
particular North-South highway in Central India or Western India will have a higher number
than the one in East India. For e.g. NH4 is somewhere in East India where as highway 44 may
be towards the west of India while both runs north-south due to the even numbering.
Similarly East-West highways will increase their numbers as we move from North to South.
For e.g. NH1 will be running East-West somewhere in North India while NH 83 may be
somewhere down south.
There may be confusion among some roads that may be running diagonally in stretches.

Q Source: National Highways

32 Wetland rice contribute up to 20% of the global CH4 emissions. Which of these bacteria may help in
microbial CH4 oxidation?

A. Escherichia coli
B. Clostridium perfringens
C. Methylobacterium
D. Campylobacter

Correct Answer : C

Answer Justification :

Learning: Due the growing human population, it has been estimated that the cultivation of this rice
will rise by 60% in the next 30 years. Therefore, CH4 emissions will need to be reduced by
microbial CH4 oxidation (as happens with Methylobacterium) or the ecosystem might become
unbalanced.

Methylobacterium species are fastidious, Gram-negative bacilli which have been reported to be
opportunistic pathogens in immunocompromised patients. These species form pink-pigmented
colonies on agar plates and have been frequently isolated from tap water in hospitals.

These are also referred as Pink-Pigmented Facultative Methylotrophs (PPFM).

These bacteria are known to metabolize methanol and some organic acids. They have the capability
to grow on one-carbon compounds such as methanol as their sole carbon and energy source. PPFMs
are aerobic bacteria that are able to grow on a wide range of multicarbon substrates.

They have beneficial effect on plants and fasten seed germination and seedling growth; accelerate
vegetative growth; and increase leaf area index and chlorophyll content.

You can read more about it in the Q Source.

Q Source: http://agritech.tnau.ac.in/technology_ppfm.htm

29
Total Marks : 200
Online Prelims TEST - 31 (TEXTBOOK)
( InsightsIAS Mock Test Series for UPSC Preliminary Exam 2020 ) Mark Scored : 0

33 Consider the following about cross-pollination.


1. In cross-pollination, the pollen moves from plant to plant carried mainly by wind or other pollinating
agents.
2. Self-pollination is not possible for a plant species outside of laboratory conditions.

A. 1 only
B. 2 only
C. Both 1 and 2
D. None of the above

Correct Answer : A

Answer Justification :

Justification: Self-pollination occurs when the pollen from the anther is deposited on the stigma of
the same flower, or another flower on the same plant. Cross-pollination is the transfer of pollen
from the anther of one flower to the stigma of another flower on a different individual of the same
species.

Cross-pollination: the pollen of one plant fertilizes the ovum of another plant of the same species
giving outcrossing or outbreeding. In cross-pollination, the pollen moves from plant to plant carried
mainly by wind for some crops or mainly by insects, often bee species, for others.

The success of cross-pollination depends on the synchrony of flowering, the longevity of pollen, and
sexual compatibility.

Cross-pollinating plants often have the male and female reproductive units on separate plants
(termed dioecious). If they are on the same plant, there is self-incompatibility in which the pollen
from that plant will not fertilize the female reproductive unit on that plant. With some outcrossing
plant species (e.g., apple and cherry), there is interspecific incompatibility between certain varieties
but other varieties are compatible. There is usually incompatibility between different species.

Q Source: Agritech Website: General

34 Consider the following with reference to livestock wellness programmes in the country.
1. Pashudhan Sanjivani has provision for Animal Health cards along with Unique Identification (UID)
of animals.
2. E-pashudhan haat portal intends to create an international market for export and import of Indian
livestock.

Which of the above is/are correct?


A. 1 only
B. 2 only
C. Both 1 and 2
D. None

30
Total Marks : 200
Online Prelims TEST - 31 (TEXTBOOK)
( InsightsIAS Mock Test Series for UPSC Preliminary Exam 2020 ) Mark Scored : 0

Correct Answer : A

Answer Justification :

Justification: Statement 1: Under the scheme 8.5 crore animals in milk will be identified using
UID and their data will uploaded in the Information Network for Animal Productivity & Health
(INAPH) data base.

This will play crucial role in control of spread of animal diseases. This will also lead to increase in
trade of livestock and livestock products.

Statement 2: At present there is no authentic market for bovine germplasm in the form of semen,
embryos, male & female calves etc. Farmers depend on middlemen for sale and purchase of quality
germplasm.

Breed wise information on availability of bovine germplasm is not available which is essential
for promotion of indigenous bovine breeds.
For the first time in the country under National Mission on Bovine Productivity E Pashudhan
Haat portal has been developed.
This portal will play important role in connecting breeders and farmers of indigenous breeds.

Q Source: http://eands.dacnet.nic.in/

35 The objectives of “India Handloom Brand” are


1. To ensure social and environmental compliances in production of handlooms
2. To earn the trust of consumers by endorsing the quality of handloom products on certain quality
parameters
3. To provide complete tax exemption to the units that qualify under the India Handloom Brand

Select the correct answer using the codes below.


A. 1 and 3 only
B. 1 and 2 only
C. 2 only
D. 1, 2 and 3

Correct Answer : B

Answer Justification :

Background: A Handloom Mark Scheme was launched by the Government of India in 2006 to
provide assurance to the consumers about authenticity of handloom products.

However, it did not cover the aspect of product quality assurance.


Therefore, the India Handloom brand is an initiative for branding of high quality handloom
products with zero defects and zero effect on the environment.

Justification: It would differentiate high quality handloom products and help in earning trust of

31
Total Marks : 200
Online Prelims TEST - 31 (TEXTBOOK)
( InsightsIAS Mock Test Series for UPSC Preliminary Exam 2020 ) Mark Scored : 0

customers by endorsing their quality in terms of raw materials, processing, embellishments,


weaving design and other quality parameters and by ensuring social and environmental
compliances in their production.

The registration for India Handloom will be granted to certain specified eligible entities in respect
of identified product categories which meet prescribed quality parameters.

Q Source: http://vikaspedia.in/social-welfare/skill-development/india-handloom-brand

36 NIDHI PRAYAS program by the Department of Science & Technology aim to harness
1. Technology incubation support to budding entrepreneurs
2. The financial corpus of higher educational institutions for industrial research

Which of the above is/are correct?


A. 1 only
B. 2 only
C. Both 1 and 2
D. None of the above

Correct Answer : A

Answer Justification :

Justification: Department of Science & Technology has launched a NIDHI program (National
Initiative for Developing and Harnessing Innovations) under which programmes for setting up of
incubators, seed fund, accelerators and ‘Proof of concept’ grant for innovators and entrepreneurs
have been launched.

Under NIDHI, PRAYAS (Promoting and Accelerating Young and Aspiring innovators & Startups)
programme has been initiated in which established Technology Business Incubators (TBI) are
supported with PRAYAS grant to support innovators and entrepreneurs with grants for ‘Proof of
Concept’ and developing prototypes.

A maximum grant of Rs. 220 lakh is given to a TBI for establishing a PRAYAS Centre which includes
Rs.100 lakh for PRAYAS SHALA, Rs. 20 lakh for operational cost of PRAYAS Centre and maximum of
Rs. 10 lakh to one innovator for developing prototype. Funding for ten innovators is given to the TBI
in a year.

A new technology has been adopted by the Maharashtra hospitals in the fight of COVID-19 fight.
The technology was developed by a Pune based Start Up.

The product is named “Scitech Airon”. It is a Negative Ion Generator.

The technology has been developed under the NIDHI PRAYAS program.

Q Source:
https://www.insightsonindia.com/2020/03/31/insights-daily-current-affairs-pib-summary-31-march-2
020/
32
Total Marks : 200
Online Prelims TEST - 31 (TEXTBOOK)
( InsightsIAS Mock Test Series for UPSC Preliminary Exam 2020 ) Mark Scored : 0

37 Acute Encephalitis Syndrome (AES) has been reported to be caused in India by

A. Nipah virus
B. Zika cirus
C. Japanese encephalitis virus (JEV)
D. All of the above

Correct Answer : D

Answer Justification :

Justification: Acute encephalitis syndrome is a basket term used for referring to hospitals, children
with clinical neurological manifestation that includes mental confusion, disorientation, convulsion,
delirium, or coma.

Meningitis caused by virus or bacteria, encephalitis (mostly Japanese encephalitis) caused by virus,
encephalopathy, cerebral malaria, and scrub typhus caused by bacteria are collectively called acute
encephalitis syndrome.

Cause of the disease:

Acute Encephalitis Syndrome (AES) is considered a very complex disease as it can be caused
by various agents including bacteria, fungi, virus and many other agents.
Viruses are the main causative agents in AES cases, although other sources such as bacteria,
fungus, parasites, spirochetes, chemicals, toxins and non-infectious agents have also been
reported over the past few decades.
Japanese encephalitis virus (JEV) is the major cause of AES in India (ranging from 5%-35%).
Nipah virus, Zika virus are also found as causative agents for AES.

Learning: The disease most commonly affects children and young adults and can lead to
considerable morbidity and mortality.

It is characterized as acute-onset of fever and a change in mental status (mental confusion,


disorientation, delirium, or coma) and/or new-onset of seizures in a person of any age at any time of
the year.

How is it related to litchi fruits? How it affects?

In India, AES outbreaks in north and eastern India have been linked to children eating unripe litchi
fruit on empty stomachs.

Unripe fruit contain the toxins hypoglycin A and methylenecyclopropylglycine (MCPG), which cause
vomiting if ingested in large quantities. Hypoglycin A is a naturally occurring amino acid found in
the unripened litchi that causes severe vomiting (Jamaican vomiting sickness), while MCPG is a
poisonous compound found in litchi seeds.

Measures needed:

33
Total Marks : 200
Online Prelims TEST - 31 (TEXTBOOK)
( InsightsIAS Mock Test Series for UPSC Preliminary Exam 2020 ) Mark Scored : 0

Increase access to safe drinking water and proper sanitation facilities.

Improve nutritional status of children at risk of JE/AES.

Preparative measures to be in place before the possible outbreaks.

Vector control.

Better awareness generation among children, parents through Anganwadi workers, ANMs etc.

Q Source:
https://www.insightsonindia.com/2020/03/31/insights-daily-current-affairs-pib-summary-31-march-2
020/

38 Scientific interest in gas hydrates is driven mainly by its potential future role as

A. Energy resource
B. Shell gas fracking
C. Aerosol applications in climate change mitigation
D. Nano-agent in reactor coolant

Correct Answer : A

Answer Justification :

Learning: Gas hydrates are naturally occurring, solid compounds containing natural gas (mainly
methane) and water. Methane gas hydrate is stable at the seafloor at water depths beneath about
500 m.

Scientific interest in gas hydrates is driven mainly by its potential future role as an energy resource,
and by the role of methane as a strong greenhouse gas and contributor to global climate change.

Further interest is linked to the geo-hazard aspect of gas hydrate occurrences (especially in the
marine environment) related to seafloor subsidence, slumps and slides. Since the time the existence
of gas hydrates was first suspected in the Blake Outer Ridge during the Deep Sea Drilling Project
(DSDP), interest in the study of marine gas hydrates has grown in several countries.

The preliminary assessment of geological condition and limited available seismic data suggests high
possibility of occurrence of large quantity of gas hydrates within the EEZ of India.

34
Total Marks : 200
Online Prelims TEST - 31 (TEXTBOOK)
( InsightsIAS Mock Test Series for UPSC Preliminary Exam 2020 ) Mark Scored : 0

Q Source: http://www.moes.gov.in/programmes/gas-hydrates

39 The main objective of e-Kranti is to accelerate e-governance across India. The key principles of e-
Kranti are?
1. ICT Infrastructure on Demand
2. Phasing out Cloud storage for enhanced security
3. Mobile First
4. Language Standardization and language Globalization

Select the correct answer using the codes below.


A. 1, 3 and 4 only
B. 1 and 3 only
C. 2, 3 and 4 only
D. 1, 2 and 4 only

Correct Answer : B

Answer Justification :

Justification: The vision is to ensure government-wide transformation by delivering all government


services electronically to citizens via integrated, interoperable systems through multiple modes.

Using e-governance to transform governance, the mission of e-Kranti is to: Make all government
services accessible to the public in their locality via Common Service Delivery outlets; ensure
efficiency, transparency and reliability in such services at affordable costs; and realise the basic
needs of common people."

The objectives of 'e-Kranti' are as follows:

To redefine NeGP with transformational and outcome oriented e-Governance initiatives.


To enhance the portfolio of citizen centric services.
To ensure optimum usage of core Information & Communication Technology (ICT).
To promote rapid replication and integration of eGov applications.
To leverage emerging technologies.
To make use of more agile implementation models.

35
Total Marks : 200
Online Prelims TEST - 31 (TEXTBOOK)
( InsightsIAS Mock Test Series for UPSC Preliminary Exam 2020 ) Mark Scored : 0

The key principles of e-Kranti are as follows:

Transformation and not Translation.


Integrated Services and not Individual Services.
Government Process Reengineering (GPR) to be mandatory in every MMP.
ICT Infrastructure on Demand.
Cloud by Default.
Mobile First.
Fast Tracking Approvals.
Mandating Standards and Protocols.
Language Localization.
National GIS (Geo-Spatial Information System).
Security and Electronic Data Preservation.

Q Source: Major government schemes: Deity Website

40 Consider the following about Madan Mohan Malaviya.


1. He never joined a legislative council in British India.
2. He shunned the activities of the Ganga Mahasabha and Hindu Mahasabha calling them ‘ultra-
nationalists’.
3. He opposed the separate electorates for Muslims under the Lucknow Pact of 1916.
4. He served as the President of the Indian National Congress (INC).

Select the correct answer using the codes below.


A. 2, 3 and 4 only
B. 3 and 4 only
C. 1, 2 and 3 only
D. 1 and 4 only

Correct Answer : B

Answer Justification :

Justification: Statement 1: He was a member of the Imperial Legislative Council from 1912 and
when in 1919 it was converted to the Central Legislative Assembly he remained its member till
1926.

Statement 2: He founded it at Haridwar in 1905. He was a member of the Hindu Mahasabha.

Statement 3: He was a moderate leader and opposed communal politics by the British. He was an
important figure in the Non-cooperation movement. However, he was opposed to the politics of
appeasement by Congress and its participation in the Khilafat movement.

Statement 4: Malaviya was the President of the Indian National Congress on two occasions (1909,
1918). He left Congress in 1934.

Q Source: Important personalities: Freedom movement

36
Total Marks : 200
Online Prelims TEST - 31 (TEXTBOOK)
( InsightsIAS Mock Test Series for UPSC Preliminary Exam 2020 ) Mark Scored : 0

41 The theory of the three Sangams establishes that, chronologically, these were

A. Successive
B. Contemporary
C. Held at a single meeting
D. Held at a single court organized by a Tamil monarch

Correct Answer : A

Answer Justification :

Learning: The Sangam period extended from roughly 400 BC to 200 AD (early Chola period before
the interregnum), when the earliest extant works of Tamil literature were written.

The traditional accounts of Iraiyanar Ahapporul mention that there were three Sangams (I, II and
III) held, which flourished for 9990 years at frequent intervals.

These were attended by over 8598 scholars. Sage Agastyar was the founding father. The
Ahapporul commentary also mentions about their successive order and the deluges occurring
during the intervals between them.

These Sangams or academies were patronized by 197 Pandyan kings. According to the tradition, of
the three successive Sangams the first two belong to prehistory. All the three were held in the
capital of the Pandyas.

As the capital was shifted from time to time, old Madurai was the headquarters of the first Sangam,
and the second academy was held at Kapatapuram. Both these centres were washed away by the
sea during successive deluges. The third Sangam was located in modern Madurai.

Q Source: https://sol.du.ac.in/mod/book/view.php?id=1611&chapterid=1637

42 Consider the following with regards to Anti-defection provisions.


1. Legislators in no circumstances can change their party without the risk of disqualification.
2. The provisions allows a party to merge with or into another party provided that at least two-thirds
of its legislators are in favour of the merger.

Select the correct answer using the codes below.


A. 1 only
B. 2 only
C. Both 1 and 2
D. None of the above

Correct Answer : B

37
Total Marks : 200
Online Prelims TEST - 31 (TEXTBOOK)
( InsightsIAS Mock Test Series for UPSC Preliminary Exam 2020 ) Mark Scored : 0

Answer Justification :

Background: The Tenth Schedule was inserted in the Constitution in 1985 by the 52nd
Amendment Act.

It lays down the process by which legislators may be disqualified on grounds of defection by the
Presiding Officer of a legislature based on a petition by any other member of the House.

Disqualification:

If a member of a house belonging to a political party:

Voluntarily gives up the membership of his political party, or


Votes, or does not vote in the legislature, contrary to the directions of his political party.
However, if the member has taken prior permission, or is condoned by the party within 15
days from such voting or abstention, the member shall not be disqualified.
If an independent candidate joins a political party after the election.
If a nominated member joins a party six months after he becomes a member of the legislature.

Justification: S1 and S2: Exceptions under the law:

Legislators may change their party without the risk of disqualification in certain
circumstances.
The law allows a party to merge with or into another party provided that at least two-thirds of
its legislators are in favour of the merger.
In such a scenario, neither the members who decide to merge, nor the ones who stay with the
original party will face disqualification.
Decision of the Presiding Officer is subject to judicial review

The decision on question as to disqualification on ground of defection is referred to the Chairman or


the Speaker of such House, and his decision is final.

The law applies to both Parliament and state assemblies.

Q Source: Basics: Polity

43 The ruling monarchs of these three chiefdoms were together called as muventars:

A. Cheras, Cholas and Pandyas


B. Satavahanas, Pallavas and Kadambas of Banavasi
C. Hoysalas, Kakatiyas and Musunuris
D. Chalukyas, Rashtrakutas and Western Chalukyas

Correct Answer : A

Answer Justification :

Justification: The ruling monarchs of three chiefdoms of the Cheras, Cholas and Pandyas together

38
Total Marks : 200
Online Prelims TEST - 31 (TEXTBOOK)
( InsightsIAS Mock Test Series for UPSC Preliminary Exam 2020 ) Mark Scored : 0

were called muventars.

The Cholas ruled over the fertile Kaveri basin with Uraiyur as its capital and the important port was
Puhar or Kaveripattinam. The most important ruler of this kingdom was Karikal.

The Pandyas ruled over the pastoral and littoral parts with Madurai as capital and Korkai as
important port. Nedunjeliyan was the most famous king.

The Cheras controlled the hilly region in the west with Vanji or Karur as its capital and Muciris as
the well known port.

The important ruler of the kingdom was Udeyinjiral. The king was called the ventan, and this was
not only the time of great kings but also great chieftains who were subordinates of the kings. These
chieftains were divided into two—velir and non-velir. There were three kinds of chiefdoms in
Tamilakam— Velir or bigger chiefs, Vedar or the biggest chiefs and Kizar who were headman of a
small village (ur) bound the kingship.

Ashokan edicts also mention the muventars.

Q Source: https://sol.du.ac.in/mod/book/view.php?id=1611&chapterid=1637

44 The world’s biggest single structure made by living organisms which is also a World Heritage site is

A. Lord Howe Island Group


B. The Sundarbans
C. Tropical Rainforest Heritage of Sumatra
D. The Great Barrier Reef

Correct Answer : D

Answer Justification :

Learning: The Great Barrier Reef is the world’s largest coral reef system composed of over 2,900
individual reefs and 900 islands stretching for over 2,300 kilometres over an area of approximately
344,400 square kilometres.

The reef is located in the Coral Sea, off the coast of Queensland, Australia.

The Great Barrier Reef can be seen from outer space and is the world’s biggest single structure
made by living organisms.

This reef structure is composed of and built by billions of tiny organisms, known as coral polyps.

It was selected as a World Heritage Site in 1981.

Q Source: Based on UPSC past year papers

39
Total Marks : 200
Online Prelims TEST - 31 (TEXTBOOK)
( InsightsIAS Mock Test Series for UPSC Preliminary Exam 2020 ) Mark Scored : 0

45 The Drugs and Cosmetics Rules, 1945, are the set of rules under The Drugs and Cosmetics Act, 1940,
which has provisions for classification of drugs into different schedules and also guidelines for
storage, sale, display. Consider the following about Schedule H1 drugs.
1. All the antibiotics have been classified under H1.
2. Schedule H1 drugs cannot be purchased based on self-medication.
3. The government has recently notified an anti-malarial drug hydroxychloroquine under Schedule H1.

Select the correct answer using the codes below.


A. 1 and 2 only
B. 2 and 3 only
C. 1 and 3 only
D. 1, 2 and 3

Correct Answer : B

Answer Justification :

Justification: Schedule H1 has been introduced through Gazette notification GSR 588 (E) dated
30-08-2013 to check the indiscriminate use of antibiotics, anti-TB and some other drugs in the
country.

The schedule contains certain 3rd and 4th generation antibiotics (not all the antibiotics), certain
habit forming drugs and anti-TB drugs.

As per government notification, these drugs are required to be sold in the country with the
following conditions:

The supply of a drug specified in Schedule H1 shall be recorded in a separate register at the
time of the supply giving the name and address of the prescriber, the name of the patient, the
name of the drug and the quantity supplied and such records shall be maintained for three
years and be open for inspection.
The drug specified in Schedule H1 shall be labelled with the symbol Rx which shall be in red
and conspicuously displayed on the left top corner of the label, and shall also be labelled with
the following words in a box with a red border:

Statement 3: The government has notified anti-malarial drug hydroxychloroquine under Schedule-
H1.

This was done in exercise of its powers conferred by Section 26B of the Drugs and Cosmetics Act,
1940 (23 of 1940).

The move is aimed at stopping misuse of the drug which has now been allowed by the government
for prophylactic use in high risk contacts of Covid-19 patients and healthcare workers treating such
patients.

Q Source:
https://www.insightsonindia.com/2020/03/28/insights-daily-current-affairs-pib-summary-28-march-2
020/

40
Total Marks : 200
Online Prelims TEST - 31 (TEXTBOOK)
( InsightsIAS Mock Test Series for UPSC Preliminary Exam 2020 ) Mark Scored : 0

46 Which of the following objective(s) inform the work of the IUCN's World Commission on Protected
Areas (WCPA)?
1. Recognise and mainstream protected areas as natural solutions to global challenges
2. Designate a global or regional trans-national buffer zone by inter-connecting protected areas for
better corridor movement of migratory species

Which of the above is/are correct?


A. 1 only
B. 2 only
C. Both 1 and 2
D. None

Correct Answer : A

Answer Justification :

Justification: WCPA works by helping governments and others plan protected areas and integrate
them into all sectors; by providing strategic advice to policy makers; by strengthening capacity and
investment in protected areas; and by convening the diverse constituency of protected area
stakeholders to address challenging issues.

It is administered by IUCN's Global Programme on Protected Areas and has over 2,000 members,
spanning 140 countries.

The objectives of the IUCN World Commission on Protected Areas for 2017-2020, aligned with the
Promise of Sydney, are to:

Catalyze and support global efforts to expand and effectively manage systems of protected
areas to achieve the Aichi Targets for halting biodiversity loss, in particular through meeting
Aichi Target 11;
Recognise and mainstream protected areas as natural solutions to global challenges, such as
climate change, land degradation, food and water security, health and well-being;
Make the case for investment in protected area systems, supported by public policy,
incentives, capacity development and sustainable funding;
Inspire all people, across generations, geography and cultures to experience and value the
wonder of nature through protected areas and to promote more support for conservation of
natural ecosystems;
Use our acknowledged role as a key global knowledge-broker and standard setter for
protected areas to support efforts to raise the standards and practices of protected area
governance and management globally.

Q Source: https://www.iucn.org/theme/protected-areas/wcpa

47 Consider the following about Ethyl Mercaptan.


1. It is a non-toxic non-flammable gas.
2. It is found naturally in small quantities in Petroleum.

41
Total Marks : 200
Online Prelims TEST - 31 (TEXTBOOK)
( InsightsIAS Mock Test Series for UPSC Preliminary Exam 2020 ) Mark Scored : 0

Select the correct answer using the codes below.


A. 1 only
B. 2 only
C. Both 1 and 2
D. None of the above

Correct Answer : B

Answer Justification :

Justification: IUPAC (International Union of Pure and Applied Chemistry) name for the chemical is
Ethanethiol. Alternate name for the substance is Ethyl Mercaptan.

It is a colorless gas or clear liquid with a distinct odor. It is highly flammable and toxic.

It occurs naturally as a minor component of petroleum. Ethanethiol is intentionally added to butane


and propane to impart an easily noticed smell to these normally odorless fuels that pose the threat
of fire, explosion, and asphyxiation. At these concentrations, ethanethiol is not harmful.

In the underground mining industry, ethanethiol or ethyl mercaptan is referred to as "stench gas".
The gas is released into mine ventilation systems to alert mine workers during an emergency. In
Ontario, mining legislation dictates that "The alarm system in an underground mine shall, consist of
the introduction into all workplaces of sufficient quantities of ethyl mercaptan gas or similar gas to
be readily detectable by all workers".

Q Source: Additional Research: UPSC Past year papers

48 Consider the following statements.


1. The first female justice in India’s Supreme Court was Fatima Beevi.
2. Presently, the Supreme Court has only one sitting female judge, Indu Malhotra.

Select the correct answer using the codes below.


A. 1 only
B. 2 only
C. Both 1 and 2
D. None of the above

Correct Answer : A

Answer Justification :

Justification: The first female justice in the court was Fatima Beevi. Appointed to the court in
1989, she became the first female judge to be a part of the Supreme court of India, and the first
Muslim woman to be appointed to any of the higher judiciaries in country.

There have been 7 more female justices in the court since then. Presently there are 3 sitting female

42
Total Marks : 200
Online Prelims TEST - 31 (TEXTBOOK)
( InsightsIAS Mock Test Series for UPSC Preliminary Exam 2020 ) Mark Scored : 0

judges out of the total 34 judges (including Chief Justice of India) in the court namely R. Banumathi,
Indu Malhotra and Indira Banerjee.

You can access both the lists here


https://en.wikipedia.org/wiki/List_of_sitting_judges_of_the_Supreme_Court_of_India and
https://en.wikipedia.org/wiki/List_of_female_judges_of_the_Supreme_Court_of_India

Q Source:

49 Overseas citizens of India (OCIs) are allowed to and enjoy parity with Non-Resident Indians (NRIs) in
which of the following respects?
1. Practicing a private profession in India
2. Registration as a voter
3. Equality of opportunity in public employment

Select the correct answer using the codes below.


A. 1 only
B. 2 and 3 only
C. 1, 2 and 3
D. 2 only

Correct Answer : A

Answer Justification :

Justification: They enjoy parity with non-resident Indians in respect of

inter-country adoption of Indian children.


Entry fees for visiting the national monuments, historical sites and museums in India.
Practicing the following professions in India, in pursuance of the provisions contained in the
relevant Acts, namely: Doctors, dentists, nurses and pharmacists etc.

They are not entitled to the following:

under article 16 of the Constitution with regard to equality of opportunity in matters of public
employment.
under article 58 of the Constitution for election as President.
under article 66 of the Constitution for election of Vice-President.
under article 124 of the Constitution for appointment as a Judge of the Supreme Court.
under article 217 of the Constitution for appointment as a Judge of the High Court.
under section 16 of the Representation of the People Act, 1950(43 of 1950) in regard to
registration as a voter.
under sections 3 and 4 of the Representation of the People Act, 1951 (43 of 1951) with regard
to the eligibility for being a member of the House of the People or of the Council of States, as
the case may be.

OCI is not to be misconstrued as 'dual citizenship'.

43
Total Marks : 200
Online Prelims TEST - 31 (TEXTBOOK)
( InsightsIAS Mock Test Series for UPSC Preliminary Exam 2020 ) Mark Scored : 0

A registered Overseas Citizen of India is granted multiple entry, multi purpose, life-long visa for
visiting India, he/she is exempted from registration with Foreign Regional Registration Officer

Q Source: http://www.mea.gov.in/overseas-citizenship-of-india-scheme.htm

50 Which of these authorities is responsible for the management of the Foreign Exchange Management
Act, 1999?

A. Department of Economic Affairs


B. Reserve Bank of India
C. Securities and Exchange Board of India (SEBI)
D. None of the above

Correct Answer : B

Answer Justification :

Justification: RBI has some important functions:

Regulator and supervisor of the financial system:

Prescribes broad parameters of banking operations within which the country's banking and
financial system functions.
Objective: maintain public confidence in the system, protect depositors' interest and provide
cost-effective banking services to the public.

Manager of Foreign Exchange

Manages the Foreign Exchange Management Act, 1999.


Objective: to facilitate external trade and payment and promote orderly development and
maintenance of foreign exchange market in India.

Issuer of currency:

Issues and exchanges or destroys currency and coins not fit for circulation.
Objective: to give the public adequate quantity of supplies of currency notes and coins and in
good quality.

Q Source: Based on past year UPSC papers

51 Laterite soil, with some manuring and irrigation, is suitable for the cultivation of
1. Cashewnut
2. Coconut
3. Rubber

Select the correct answer using the codes below.


A. 1 and 2 only

44
Total Marks : 200
Online Prelims TEST - 31 (TEXTBOOK)
( InsightsIAS Mock Test Series for UPSC Preliminary Exam 2020 ) Mark Scored : 0

B. 3 only
C. 1 only
D. 1, 2 and 3

Correct Answer : D

Answer Justification :

Justification: The laterite soils develop in areas with high temperature and high rainfall. These are
the result of intense leaching due to tropical rains - summits of Western Ghats at 1000 to 1500 m
above mean sea level, Eastern Ghats, the Rajmahal Hills, Vindhyan, Satpuras and Malwa Plateau.

With rain, lime and silica are leached away, and soils rich in iron oxide and aluminium compound
are left behind. Humus content of the soil is removed fast by bacteria that thrives well in high
temperature.

These soils are poor in organic matter, nitrogen, phosphate and calcium, while iron oxide and
potash are in excess.

Hence, laterites are not suitable for cultivation; however, application of manures and fertilisers are
required for making the soils fertile for cultivation.

Red laterite soils in Tamil Nadu, Andhra Pradesh and Kerala are more suitable for tree crops like
cashewnut.

Some laterites are also suitable for growing plantation crops like tea, coffee, rubber, cinchona,
coconut, arecanut, etc.

In some areas, these soils support grazing grounds and scrub forests.

45
Total Marks : 200
Online Prelims TEST - 31 (TEXTBOOK)
( InsightsIAS Mock Test Series for UPSC Preliminary Exam 2020 ) Mark Scored : 0

Q Source:11th NCERT: India Physical Geography

52 Consider the following with regards to border haats.


1. The trade at border haats is permitted to be carried out in both currency and barter basis.
2. Sale of any forest produce is not allowed at the haats to curb cross-border smuggling.
3. Presently, the only border haats operational in India are located in Assam.

Select the correct answer using the codes below.


A. 1 only
B. 2 and 3 only
C. 1, 2 and 3
D. 1 and 2 only

Correct Answer : A

46
Total Marks : 200
Online Prelims TEST - 31 (TEXTBOOK)
( InsightsIAS Mock Test Series for UPSC Preliminary Exam 2020 ) Mark Scored : 0

Answer Justification :

Justification: Statement 1:The trade at border haats is permitted to be carried out in Indian
Rupees/Bangladesh Taka and on a barter basis, and data of such trade is maintained by the Haat
Management Committee of the respective border haat.

Statement 3: Border Haatsare market places organised by the two countries one day each week. It
is not only a market for buying daily commodities but also a reunion spot for families living on both
sides.

The border haats aim at promoting the wellbeing of the people dwelling in remote areas across the
borders of two countries, by establishing traditional system of marketing the local produce through
local markets. Operational Border Haats between India and Bangladesh:

Currently, four border haats are operational along the India-Bangladesh border. Two border haats
are located in Meghalaya at Kalaichar and Balat and two are located in Tripura at Srinagar and
Kamalasagar.

Statement 2:The Following commodities are traded in the Border Haats:

Vegetables, food items, fruits, spices.

Minor Forest produceeg bamboo, bamboo grass and broom sticks, excluding timber.

Products of Cottage Industries like Gamcha, Lungi etc.

Small Agriculture household implements egdao, plough, axe, spade, chisel etc.

Garments, melamine products, processed food items, fruit juice, toiletries, cosmetics, plastic
products, aluminium products, cookeries.

Q
Source:https://www.insightsonindia.com/wp-content/uploads/2020/04/INSTA-PT-2020-Exclusive-Int
ernational-Relations.pdf

53 . What is/are the benefits of signing Agreement on Reciprocal Logistics Support (ARLS) for India and
Russia?
1. It will allow access to India and Russia, to each other’s military facilities for supplies and fuel.
2. India will gain access to energy resource rich ports in the Russian part of the Arctic.

Consider the following with regards to Select the correct answer using the codes below.
A. 1 only
B. 2 only
C. Both 1 and 2
D. None

Correct Answer : C

47
Total Marks : 200
Online Prelims TEST - 31 (TEXTBOOK)
( InsightsIAS Mock Test Series for UPSC Preliminary Exam 2020 ) Mark Scored : 0

Answer Justification :

Justification: Agreement on Reciprocal Logistics Support (ARLS)

India and Russia are finalising a defence agreement that will simplify interoperability and enable
military platforms to receive support and supplies across bases in both nations-

It is an arrangement that will allow access to India and Russia, to each other’s military facilities for
supplies and fuel, expanding the logistics support and operational turnaround of the Indian military.

Benefits and mutual significance: This will be beneficial for the Indian Navy, which has a large
number of Russian origin ships, that will get access to Russian ports for supplies and refueling. It
would be crucial for joint exercises.

The air force too will benefit by finding it easier to deploy aircraft for the same purpose. This access
will also be for ports in the Russian part of the Arctic, allowing access to energy resources there.
Russia, on the other hand, will be able to access Indian ports and air bases. Russia has also assured
India access to energy resources in the vast Arctic region

Q
Source:https://www.insightsonindia.com/wp-content/uploads/2020/04/INSTA-PT-2020-Exclusive-Int
ernational-Relations.pdf

54 Consider the following statements.


Assertion (A): Current Account Deficit (CAD) as a percentage of GDP has consistently declined year-
on-year since 2012 Eurozone Slowdown.
Reason (R): India’s trade balance (export minus imports) as a percentage of GDP has consistently
increased year-on-year since 2012.

In the context of the above, which of these is correct?


A. A is correct, and R is an appropriate explanation of A.
B. A is correct, but R is not an appropriate explanation of A.
C. A is correct, but R is incorrect.
D. Both A and R are incorrect.

Correct Answer : D

Answer Justification :

Justification:The charts below depict that both A and R are incorrect. The trend has been erratic
for the last decade or so.

48
Total Marks : 200
Online Prelims TEST - 31 (TEXTBOOK)
( InsightsIAS Mock Test Series for UPSC Preliminary Exam 2020 ) Mark Scored : 0

Q Source:Ch 1: Vol2 : ES19-20

55 Global Social Mobility Report was recently released by

A. International Monetary Fund


B. World Bank
C. United Nations Economic and Social Council
D. World Economic Forum

Correct Answer : D

Answer Justification :

Justification: World Economic Forum has come out with its first-ever Global Social Mobility
Report. The report has ranked India a lowly 76 out of the 82 countries profiled.

What is social mobility?

49
Total Marks : 200
Online Prelims TEST - 31 (TEXTBOOK)
( InsightsIAS Mock Test Series for UPSC Preliminary Exam 2020 ) Mark Scored : 0

It can be understood as the movement in personal circumstances either “upwards” or “downwards”


of an individual in relation to those of their parents.

In absolute terms, it is the ability of a child to experience a better life than their parents. On the
other hand, relative social mobility is an assessment of the impact of socioeconomic background on
an individual’s outcomes in life. It can be measured against a number of outcomes ranging from
health to educational achievement and income.

Q Source:Insights Current affairs

56 The Governor is authorised to


1. Direct that any particular Act of Parliament or of the Legislature of the State shall not apply to a
Scheduled Area or apply with modifications
2. Make regulations to prohibit or restrict the transfer of land by or among members of the Scheduled
Tribes

Which of the above is/are correct?


A. 1 only
B. 2 only
C. Both 1 and 2
D. None

Correct Answer : C

Answer Justification :

Justification and Learning: Statement 1: This is done in view of the unique socio-cultural dynamic
of tribal societies that is not amenable to the single stroke legal treatment that flows unilaterally
from the Centre. The Governor’s discretion adds some legal flexibility.

Statement 2: The Governor may make regulations for the peace and good government of any area in
a State which is for the time being a Scheduled Area.

These provisions of the Constitution relating to the administration of the Scheduled Areas and
Tribes may be altered by Parliament or by ordinary legislation.

The Constitution provides for the appointment of a Commission to report on the administration of
the Scheduled Areas and the welfare of the Scheduled Tribes in the States.

The President may appoint such Commission at any time, but the appointment of such Commission
at the end of 10 years from the commencement of the Constitution was obligatory.

Q Source: Chapter on Scheduled areas: Indian Polity: M Laxmikanth

57 Consider the following about the Group of 77 (G77).


1. It is a group of 77 developing countries.
2. India and China are members of G77

50
Total Marks : 200
Online Prelims TEST - 31 (TEXTBOOK)
( InsightsIAS Mock Test Series for UPSC Preliminary Exam 2020 ) Mark Scored : 0

3. It was established with a primary focus on climate change mitigation.

Consider the following with regards to Select the correct answer using the codes below.
A. 1 only
B. 1, 2 and 3
C. 1 and 2 only
D. None of the above

Correct Answer : D

Answer Justification :

Justification: The Group of 77 (G77) at the United Nations is a coalition of 134 developing nations,
designed to promote its members’ collective economic interests and create an enhanced joint
negotiating capacity in the United Nations.

G77 was formed on 15 June 1964 by the “Joint Declaration of the Seventy-Seven Countries” issued
at the United Nations Conference on Trade and Development (UNCTAD).

Since China participates in the G77 but does not consider itself to be a member, all official
statements are issued in the name of The Group of 77 and China.

Egypt held the Chairmanship for 2018. Palestine was the chair of the group since January 2019 and
Guyana holds the chairmanship as of 2020.

The chairmanship of the G77 is based on the system of geographical rotation

Q Source:Sometimes in news

58 Which of the following is/are mature Harappan sites?


1. Amri
2. Mitathal
3. Chanhudaro
4. Manda

Select the correct answer using the codes below.


A. 1 and 2 only
B. 3 only
C. 3 and 4 only
D. 1, 2, 3 and 4

Correct Answer : D

Answer Justification :

Learning: Please refer to the NCERT map below.

51
Total Marks : 200
Online Prelims TEST - 31 (TEXTBOOK)
( InsightsIAS Mock Test Series for UPSC Preliminary Exam 2020 ) Mark Scored : 0

Q Source: Chapter 1: 12th NCERT: Themes in Indian History: Part-I

59 The proceedings of the UN Commission on International Trade Law (UNCITRAL) are administered by

A. World Trade Organization (WTO)


B. Permanent Court of Arbitration, Hague
C. United Nations Economic and Social Council (UNECOSOC)
D. World Customs Organization (WCO)

Correct Answer : B

Answer Justification :

Justification: The UN Commission on International Trade Law (UNCITRAL) is a subsidiary body of


the U.N. General Assembly.

It is responsible for helping to facilitate international trade and investment. Established by the
UNGA in 1966. Mandate is “to promote the progressive harmonization and unification of
international trade law” through conventions, model laws, and other instruments that address key
areas of commerce, from dispute resolution to the procurement and sale of goods.

The Tribunal was constituted in accordance with the UNCITRAL Arbitration Rules 1976 is seated at
the Hague, Netherlands, and proceedings are administered by the Permanent Court of
Arbitration.

Annual sessions held alternately in New York City and Vienna, where it is headquartered.

Q Source:Insights current affairs

52
Total Marks : 200
Online Prelims TEST - 31 (TEXTBOOK)
( InsightsIAS Mock Test Series for UPSC Preliminary Exam 2020 ) Mark Scored : 0

60 Consider the following with regards to the Vindhyan ranges.


1. The Vindhyan Mountains form a dividing line between the Ganges plain and Deccan Plateau.
2. Vindhyan system rocks are extensively distributed in Eastern and Western India, including the
states of Rajasthan and Bihar.
3. The well known Panna and Golconda diamonds are found in the Vindhyan formation.

Select the correct answer using the codes below.


A. 1 only
B. 1 and 2 only
C. 2 and 3 only
D. 1, 2 and 3

Correct Answer : D

Answer Justification :

Justification: The Vindhyan Mountains form a dividing line between the Ganges plain and Deccan
Plateau.

This system rocks are extensively distributed in India from Chittorgarh in Rajasthan to Sasaram in
Bihar.

The well known Panna and Golconda diamonds are found in this formation. The important series of
this system are Bhander series, Bijwar series and Kaimur series. All are rich sources of Building
material.

The rock systems of India (dharwar, archean, cuddapah, vindhyan, tertiary) are very important. We
have covered some of them in previous tests. Do read about them in detail.

Q Source: Additional Research: UPSC Past year papers

61 Match the following places in news with the country they are located in:
1. Idlib: Iraq
2. Kalapani: Nepal
3. Bhashan Char: Bangladesh

Select the correct answer using the codes below.


A. 2 only
B. 1 and 3 only
C. 2 and 3 only
D. 1, 2 and 3

Correct Answer : C

Answer Justification :

53
Total Marks : 200
Online Prelims TEST - 31 (TEXTBOOK)
( InsightsIAS Mock Test Series for UPSC Preliminary Exam 2020 ) Mark Scored : 0

Justification: Statement 1: Idlib: The nine-year-old war in Syria is currently raging in the
northwestern province of Idlib, with rapidly escalating tensions between government forces of
President Bashar al-Assad and the Turkish military. Idlib is a city in northwestern Syria, 59
kilometers southwest of Aleppo, which is the capital of the Idlib Governorate.

Statement 2:Nepal and India are planning to resolve the Kalapani border issue through dialogue.

In the latest map, India included Kalapani into the Pithoragarh district of Uttarakhand. Nepal
government says that Kalapani is an integral part of the country and that talks are still on between
New Delhi and Kathmandu over this “unresolved” area.

Where is it located?

Kalapani is located at an altitude of 3600m on the Kailash Manasarovar route. It borders


Uttarakhand in India and Sudurpashchim Pradesh in Nepal. Since the Indo-China war of 1962,
Kalapani is controlled by India’s Indo-Tibetan Border Police. Nepal claims that the river located
towards the west of the territory is the main Kali river and thus it falls in its territory, India claims a
ridgeline towards the east of the Kalapani territory and hence, includes it in the Indian Union.

Under the treaty of Sugauli signed between Nepal and the British East India Company in 1816, the
Kali River was located as Nepal’s western boundary with India. It, however, made no mention of a
ridgeline and subsequent maps of the areas drawn by British surveyors showed the source of the
Kali river at different places.

Statement 3:The Rohingya, who numbered around one million in Myanmar at the start of 2017, are
one of the many ethnic minorities in the country. Rohingya Muslims represent the largest
percentage of Muslims in Myanmar, with the majority living in Rakhine state.

Rohingyas will be relocated to the Bhashan Char (Bangladesh). Authorities concerned, especially
the Bangladesh Navy, have prepared the Bhashan Char for rehabilitation of at least one lakh
Rohingyas. Location: Bhashan Char is located in the estuary of the Meghna river. It falls in an
ecologically fragile area prone to floods, erosion and cyclone.

Q Source:Insights current affairs: Places in news

62 INTERPOL’s declared policing goals include


1. Providing a secure cyberspace for people and businesses
2. Curbing illicit markets
3. Supporting environment security

Select the correct answer using the codes below.


A. 2 only
B. 1 and 3 only
C. 1, 2 and 3
D. 1 only

Correct Answer : C

54
Total Marks : 200
Online Prelims TEST - 31 (TEXTBOOK)
( InsightsIAS Mock Test Series for UPSC Preliminary Exam 2020 ) Mark Scored : 0

Answer Justification :

Justification: The International Criminal Police Organisation, or Interpol, is a 194-member


intergovernmental organisation. Headquartered in Lyon, France.

Formed in 1923 as the International Criminal Police Commission, and started calling itself Interpol
in 1956. Interpol’s declared global policing goals include countering terrorism, promoting border
integrity worldwide, protection of vulnerable communities, providing a secure cyberspace for
people and businesses, curbing illicit markets, supporting environment security, and promoting
global integrity. India joined the organisation in 1949.

Learning: What is an Interpol notice?

Notices are international requests for cooperation or alerts allowing police in member countries to
share critical crime-related information. There are seven types of notices — Red Notice, Yellow
Notice, Blue Notice, Black Notice, Green Notice, Orange Notice, and Purple Notice. What is blue
notice? Issued to “collect additional information about a person’s identity, location or activities in
relation to a crime.”

Q Source:Based on previous test topics

63 Which of the following festivals is the largest congregation of women for a festival in the world?

A. GudiPadwa
B. AttukalPongala
C. Rath Yatra
D. KarthikaiDeepam

Correct Answer : B

Answer Justification :

Justification: The Kuthiyottam ritual is usually performed every year during the Pongala festival at
the AttukalBhagavathy Temple in Thiruvananthapuram, Kerala.

Pongala, which means ‘to boil over’, is a ritual in which women prepare a pudding made from rice,
jaggery, coconut and plantains cooked together, and offer it to the goddess.

The ritual can only be performed by women.

Learning: The Kerala State Commission for the Protection of Child Rights has registered a suo
motu case in connection with the Kuthiyottam ritual.

The commission said it would examine if the ritual, reportedly involving piercing children’s
sides with a hook, violated child rights in any manner.
Nearly 1,000 young boys undertake a seven-day penance before Pongala day.
These boys are said to represent the wounded soldiers of the goddess.
The boys have to observe strict discipline and stay inside the temple for seven days.

55
Total Marks : 200
Online Prelims TEST - 31 (TEXTBOOK)
( InsightsIAS Mock Test Series for UPSC Preliminary Exam 2020 ) Mark Scored : 0

The rigours include sleeping on the floor, strict diet restrictions, and bathing three times a
day.
They also have to prostrate 1,008 times before the deity.
The ritual also reportedly involves piercing the child’s side with a small hook and knotting a
thread through it to symbolise their bond with the Goddess.

Q Source:
http://www.guinnessworldrecords.com/world-records/largest-annual-gathering-of-women/

64 India’s seed vault, to protect important genetic material in case of a major man-made or natural
disaster, is located in

A. Kanchendzonga
B. Karakoram Pass
C. Ladakh
D. Nanda Devi

Correct Answer : C

Answer Justification :

Justification: Paralleling the Arctic Seed Vault, the ICAR and DRDO have established an Indian
seed vault. India’s seed vault is located in Chang La (Ladakh) in the Himalayas, at a height of
17,300 feet. There is a storage facility with over 5,000 seed accessions.

The vault is a joint venture of the National Bureau of Plant Genetic Resources (which comes under
the Indian Council of Agricultural Research) and the Defence Institute of High Altitude Research
(under Defence Research and Development Organisation).

Learning: Norway has announced it will spend 100 million Norwegian crowns, or $13 million, to
make improvements to a special seed vault in the Arctic.

It has the ability to hold about 2.5 billion seeds in total.


The Norwegian government built the Svalbard Global Seed Vault in 2008 to store seed
samples of the world’s crops and plants.
The vault serves as a natural deep freeze, and is meant to protect important genetic material
in case of a major man-made or natural disaster.
The loss of crop diversity and the vulnerability of existing seed collections in various
seed/gene banks gave birth to the idea in of establishing a back-up seed facility in Svalbard.
Depositors retain ownership rights over the seeds sent to the facility. The boxes with seeds
are sealed by the depositors and are not distributed to or given access to by anyone other
than the depositors.
The cold climate and permafrost of Svalbard make the area a perfect location for
underground cold storage.
The surrounding sandstone is stable for building and is low in radiation.
In terms of security, Svalbard scores high marks compared to the locations of many other
gene banks in the world.

56
Total Marks : 200
Online Prelims TEST - 31 (TEXTBOOK)
( InsightsIAS Mock Test Series for UPSC Preliminary Exam 2020 ) Mark Scored : 0

The vault is located an extraordinary 120 meters into the rock, ensuring that the vault rooms will
remain naturally frozen even in the event of failure of the mechanical cooling system and rising
external air temperatures due to climate change.

Q Source: Based on past year UPSC papers

65 Countervailing duty is imposed on imports to


1. Counter the impact of import subsidies on domestic producers
2. Raise the price of goods that were ‘dumped’ by sellers abroad in domestic market

Which of the above is/are correct?


A. 1 only
B. 2 only
C. Both 1 and 2
D. None of the above

Correct Answer : A

Answer Justification :

Learning:S1:Duties that are imposed in order to counter the negative impact of import subsidies to
protect domestic producers are called countervailing duties.

Therefore, Countervailing duty is a customs duty on goods that have received government subsidies
in the originating or exporting country.

S2: An anti-dumping duty (ADD) is a customs duty on imports providing a protection against the
dumping of goods in the EU at prices substantially lower than the normal value. In most cases, this
is the price foreign producers charge for comparable sales in the producer’s own country. It is
possible to have both ADD and countervailing duty on a product, but they are different
duties.

ADD is chargeable in addition to, and independent of, any other duty to which the imported goods
are liable.

Explanation of CD: In cases foreign producers attempt to subsidize the goods being exported by
them so that it causes domestic production to suffer because of a shift in domestic demand towards
cheaper imported goods, the government makes mandatory the payment of a countervailing duty on
the import of such goods to the domestic economy.

This raises the price of these goods leading to domestic goods again being equally competitive and
attractive. Thus, domestic businesses are cushioned.

Q Source:Based on past year UPSC papers

66 Which of the following Indian Particularly Vulnerable Tribal Groups (PVTGs) are found only on
islands?

57
Total Marks : 200
Online Prelims TEST - 31 (TEXTBOOK)
( InsightsIAS Mock Test Series for UPSC Preliminary Exam 2020 ) Mark Scored : 0

1. Toto
2. Onges
3. Shompen

Select the correct answer using the codes below.


A. 1 only
B. 2 and 3 only
C. 1, 2 and 3
D. 1 and 3 only

Correct Answer : B

Answer Justification :

Justification: PVTGs are only found in one island chain in India - A&N islands. We provide a brief
overview of all major tribes of A&N.

The Andaman Islands are home to four 'Negrito' tribes – the Great Andamanese, Onge, Jarawa and
Sentinelese. The Nicobar Islands are home to two 'Mongoloid' tribes – the Shompen and
Nicobarese.

The 'Negrito' tribes are believed to have arrived in the islands from Africa up to 60,000 years ago.
All are nomadic hunter-gatherers, hunting wild pig and monitor lizard, and catching fish with bows
and arrows. They also collect honey, roots and berries from the forest.

The 'Mongoloid' tribes probably came to the islands from the Malay-Burma coast several thousand
years ago.

Great Andamanese: The tribe that has suffered most from contact with outsiders: 99% have been
wiped out since the British first colonized the islands. Before the tsunami they lived in a
government settlement and were dependent on government aid. They have just been moved to the
islands' capital, Port Blair, as their village was badly damaged in the tsunami.

Onge: Their forest home has been plundered by poachers and loggers. They were settled by the
Indian administration and are dependent on food handouts. The Onge living at Dugong Creek fled to
high ground when they saw the sea level fall, and so survived.

Jarawa: Have only had peaceful contact with outsiders for six years. They live on the west coast of
South and Middle Andaman, and are believed to have survived the tsunami. They are still
completely independent and live entirely by hunting, gathering and fishing. The main threat to their
existence comes from the highway running through their territory: the Indian government was
ordered to close this by the Supreme Court in 2002, but it has ignored the order.

Sentinelese: The most isolated of all the tribes, they have no peaceful contact with outsiders, and
fire warning arrows at those who approach. Their home, Sentinel Island, appears to have been
relatively unaffected by the tsunami, and some Sentinelese have been sighted since the disaster.
They are completely self-sufficient hunter-gatherers

58
Total Marks : 200
Online Prelims TEST - 31 (TEXTBOOK)
( InsightsIAS Mock Test Series for UPSC Preliminary Exam 2020 ) Mark Scored : 0

Shompen: A relatively isolated tribe of Great Nicobar Island, the Shompen are hunter-gatherers
who have some, limited, contact with outsiders. Overflights of their territory suggest their forest
has been little damaged, raising hopes that the tribe has survived more or less intact.

Q Source:https://tribal.nic.in/DivisionsFiles/ListofPVTGs191212.pdf

67 Which of these Ministries of the Government of India, that has the most number of Central Sector
Public Enterprises (CPSEs) under it, also has the most number of profitable CPSEs amongst all its
CPSEs?

A. Ministry of Defence
B. Ministry of Steel
C. Ministry of Commerce and Industry
D. Ministry of Power

Correct Answer : D

Answer Justification :

Justification:

Q Source:Ch 9: Economic Survey (ES) 19-20

68 Consider the following about Innovate in India (i3) programme.


1. It is a Government-Industry partnership to search for innovative ideas from rural grassroots and
provide them with mentorship and incubation support
59
Total Marks : 200
Online Prelims TEST - 31 (TEXTBOOK)
( InsightsIAS Mock Test Series for UPSC Preliminary Exam 2020 ) Mark Scored : 0

2. It was established on the behest of the World Bank (WB) and supported by the International
Chambers of Commerce.

Which of the above is/are correct?


A. 1 only
B. 2 only
C. Both 1 and 2
D. None

Correct Answer : D

Answer Justification :

Justification: Towards strengthening the emerging biotechnology enterprise in India, Department


of Biotechnology (DBT), Ministry of Science & Technology has initiated the Mission programme
entitled: Industry-Academia Collaborative Mission for Accelerating Discovery Research to Early
Development for Biopharmaceuticals – “Innovate in India (i3) Empowering biotech entrepreneurs &
accelerating inclusive innovation”.

This Mission of the Department was approved and formally launched in 2017.

S2: The Mission has been approved at a total cost of US$250 million for five years with 50% funding
through World Bank Loan, with an aim to make India a hub for design and development of novel,
affordable and effective biopharmaceutical products such as vaccines, biologics and medical devices
for combating public health concerns. It wasn’t established by WB or formally supported by the ICC.

This Programme of DBT would strengthen translational capability of academic researchers;


empower bio-entrepreneurs and SMEs by decreasing the cost and risk during early stages of
product development and also elevate the innovation quotient of the industry.
The global experience of World Bank would be instrumental in building sustained global
linkages, technical assistance and knowledge flow between public private partners for
business promotion in biotech sector.

Q Source: http://www.dbtindia.nic.in/innovate-in-india-i3-signing-of-legal-agreements/

69 The countries that have a larger GDP (Nominal) than India is/are
1. Germany
2. United Kingdom
3. France

Consider the following with regards to Select the correct answer using the codes below.
A. 2 only
B. 1 and 3 only
C. 1 and 2 only
D. 1 only

60
Total Marks : 200
Online Prelims TEST - 31 (TEXTBOOK)
( InsightsIAS Mock Test Series for UPSC Preliminary Exam 2020 ) Mark Scored : 0

Correct Answer : D

Answer Justification :

Justification: Please refer to the table below from the survey.

Q Source:Ch 1: Volume 2: ES19-20

70 Big Data Analytics can be used for which of the following applications?
1. Understanding customer preferences online
2. Planninga large smart city
3. Analysis of Internet Traffic

Select the correct answer using the codes below.


A. 1 and 2 only
B. 1 and 3 only
C. 1, 2 and 3
D. 2 only

Correct Answer : C

Answer Justification :

Justification: Big data analytics is the process of examining large data sets containing a variety of
data types -- i.e., big data -- to uncover hidden patterns, unknown correlations, market trends,
customer preferences and other useful business information.

The analytical findings can lead to more effective marketing, new revenue opportunities, better
customer service, improved operational efficiency, competitive advantages over rival organizations
and other business benefits.

61
Total Marks : 200
Online Prelims TEST - 31 (TEXTBOOK)
( InsightsIAS Mock Test Series for UPSC Preliminary Exam 2020 ) Mark Scored : 0

For e.g. Madhya Pradesh sometime before established India's first Smart Cities Control Centre that
would make use of big data to improve city management, catch traffic offenders, reduce congestion
and provide better civic amenities.

Q Source: Often in news

71 Consider the following statements.


1. Biochemical pesticides are naturally occurring substances that control pests by non-toxic
mechanisms.
2. Canola oil and Baking soda are considered biopesticides.
3. Bacterium, fungus, virus or protozoan all can be used as biopesticide.

Select the correct answer using the codes below.


A. 3 only
B. 1, 2 and 3
C. 1 and 3 only
D. 2 only

Correct Answer : B

Answer Justification :

Justification: Biopesticides are certain types of pesticides derived from such natural materials as
animals, plants, bacteria, and certain minerals. For example, canola oil and baking soda have
pesticidal applications and are considered biopesticides. As of April 2016, there are 299 registered
biopesticide active ingredients and 1401 active biopesticide product registrations.

Biopesticides fall into three major classes:

1. Biochemical pesticides are naturally occurring substances that control pests by non-toxic
mechanisms. Conventional pesticides, by contrast, are generally synthetic materials that
directly kill or inactivate the pest. Biochemical pesticides include substances that interfere
with mating, such as insect sex pheromones, as well as various scented plant extracts that
attract insect pests to traps. Because it is sometimes difficult to determine whether a
substance meets the criteria for classification as a biochemical pesticide, EPA has established
a special committee to make such decisions.
2. Microbial pesticides consist of a microorganism (e.g., a bacterium, fungus, virus or
protozoan) as the active ingredient. Microbial pesticides can control many different kinds
of pests, although each separate active ingredient is relatively specific for its target pest[s].
For example, there are fungi that control certain weeds and other fungi that kill specific
insects.

The most widely used microbial pesticides are subspecies and strains of Bacillus
thuringiensis, or Bt. Each strain of this bacterium produces a different mix of proteins and
specifically kills one or a few related species of insect larvae. While some Bt ingredients
control moth larvae found on plants, other Bt ingredients are specific for larvae of flies and
mosquitoes. The target insect species are determined by whether the particular Bt produces

62
Total Marks : 200
Online Prelims TEST - 31 (TEXTBOOK)
( InsightsIAS Mock Test Series for UPSC Preliminary Exam 2020 ) Mark Scored : 0

a protein that can bind to a larval gut receptor, thereby causing the insect larvae to starve.

1. Plant-Incorporated-Protectants (PIPs) are pesticidal substances that plants produce from


genetic material that has been added to the plant. For example, scientists can take the gene
for the Bt pesticidal protein and introduce the gene into the plant's own genetic material.
Then the plant, instead of the Bt bacterium, manufactures the substance that destroys the
pest. The protein and its genetic material, but not the plant itself, are regulated by EPA.

Q Source:https://www.epa.gov/ingredients-used-pesticide-products/what-are-biopesticides

http://www.dbtindia.nic.in/related-links/

72 These state(s) do not have scheduled areas, but have nevertheless constituted Tribe Advisory Council
(TAC) for certain areas:
1. West Bengal
2. Rajasthan
3. Uttarakhand
4. Tamil Nadu

Select the correct answer using the codes below.


A. 1 and 3 only
B. 1 only
C. 2, 3 and 4 only
D. 1, 3 and 4 only

Correct Answer : D

Answer Justification :

Justification: Tribes Advisory Councils (TAC) have been constituted in Scheduled Area States of
Andhra Pradesh, Chhattisgarh, Gujarat, Jharkhand, Himachal Pradesh, Madhya Pradesh,
Maharashtra, Odisha, Rajasthan and Telangana. Though Tamil Nadu, Uttarakhand and West Bengal
do not have any scheduled area, they have also constituted TAC.

Learning:The para 4 of the Fifth Schedule to the Constitution envisages as under:- 4. Tribes
Advisory Council.—(1) There shall be established in each State having Scheduled Areas therein and,
if the President so directs, also in any State having Scheduled Tribes but not Scheduled Areas
therein, a Tribes Advisory Council consisting of not more than twenty members of whom, as nearly
as may be, three-fourths shall be the representatives of the Scheduled Tribes in the Legislative
Assembly of the State: (2) It shall be the duty of the Tribes Advisory Council to advise on such
matters pertaining to the welfare and advancement of the Scheduled Tribes in the State as may be
referred to them by the Governor.”

Q Source:https://tribal.nic.in/DivisionsFiles/clm/TribesAdvisoryCouncil.pdf

73 Global Tiger recovery programme (GTRP) is an initiative of


⌰〰〰

63
Total Marks : 200
Online Prelims TEST - 31 (TEXTBOOK)
( InsightsIAS Mock Test Series for UPSC Preliminary Exam 2020 ) Mark Scored : 0

A. UNEP
B. IUCN
C. WWF
D. Conservation International

Correct Answer : C

Answer Justification :

Learning: The overarching goal adopted in the Hua Hin declaration and supported by the Global
Tiger Recovery Program (GTRP) is to reverse the rapid decline of wild tigers and to strive to double
the number of wild tigers across their range by 2022.

The Tiger Range Country's (TRCs) as a group is ready to take on this challenge although not all
TRCs individually will be able to achieve this goal.

To solve the tiger crisis, which represents the larger Asian biodiversity crisis, the TRCs,
international organizations, and civil society have come together on a collaborative platform within
the framework of the Global Tiger Initiative (GTI).

After a two-year process of sharing knowledge and best practices and developing a common vision,
the GTRP was developed, with the shared goal of doubling the number of wild tigers globally by
2022 through actions to:

effectively manage, preserve, protect, and enhance tiger habitats;


eradicate poaching, smuggling, and illegal trade of tigers, their parts, and derivatives;
cooperate in transboundary landscape management and in combating illegal trade;
engage with indigenous and local communities;
increase the effectiveness of tiger and habitat management; and
restore tigers to their former range.

Q Source: http://tigers.panda.org/tx2/

http://documents.worldbank.org/curated/en/874191468331048098/Global-tiger-recovery-program-2
010-2022

74 National Dairy Plan Phase I (NDP I) was a Central Sector Scheme with which of the following
objectives?
1. To help increase productivity of milch animals
2. To help provide rural milk producers with greater access to the organised milk-processing sector

Select the correct answer using the codes below.


A. 1 only
B. 2 only
C. Both 1 and 2
D. None of the above

64
Total Marks : 200
Online Prelims TEST - 31 (TEXTBOOK)
( InsightsIAS Mock Test Series for UPSC Preliminary Exam 2020 ) Mark Scored : 0

Correct Answer : C

Answer Justification :

Justification: National Dairy Plan Phase I (NDP I) was a Central Sector Scheme for a period of
2011-12 to 2018-19. NDP was implemented with a total investment of about ₹ 2242 crore
comprising ₹ 1584 crore as International Development Association (IDA) credit, ₹ 176 crore as
Government of India share, ₹ 282 crore as share of End Implementing Agencies (EIAs) that will
carry out the projects in participating states and ₹ 200 crore by National Dairy Development Board
and its subsidiaries for providing technical and implementation support to the project.

Funding was through a line of credit from the International Development Association (IDA), which
along with the share of the Government of India will flow from DADF to NDDB and in turn to
eligible EIAs.

​NDP I is a scientifically planned multi-state initiative with the following Project Development
Objectives :

To help increase productivity of milch animals and thereby increase milk production to meet
the rapidly growing demand for milk
To help provide rural milk producers with greater access to the organised milk-processing
sector

These objectives would be pursued through adoption of focused scientific and systematic processes
in provision of technical inputs supported by appropriate policy and regulatory measures

NDP I will focus on 18 major milk producing states namely Andhra Pradesh, Bihar, Gujarat,
Haryana, Karnataka, Kerala, Madhya Pradesh, Maharashtra, Odisha, Punjab, Rajasthan, Tamil
Nadu, Uttar Pradesh, West Bengal, Telangana, Uttarakhand, Jharkhand and Chhattisgarh which
together account for over 90% of the country’s milk production. Coverage of NDP I will however be
across the country in terms of benefits accruing from the scheme.

Q Source:Department of Animal Husbandry

75 PACEsetter Fund programme concerns the development of

A. Medical devices
B. Innovative clean energy products
C. Mobility solutions
D. Water purification technologies

Correct Answer : B

Answer Justification :

Justification: PACEsetter fund: Constituted by India and the USA in 2015. It is a INR 50 crore

65
Total Marks : 200
Online Prelims TEST - 31 (TEXTBOOK)
( InsightsIAS Mock Test Series for UPSC Preliminary Exam 2020 ) Mark Scored : 0

(USD 7.9 million) fund jointly capitalized by the Governments of the Republic of India and the
United States of America. It is a joint fund to provide early-stage grant funding to accelerate the
commercialization of innovative off- grid clean energy products, systems, and business models.

The Fund will award grants for direct support of innovative technology, business models, and
programs including but not limited to: rural energy services companies (full scale integrated
operators); rural distribution companies/franchisees; operations/maintenance companies;
technology implementers/system integrators; and enterprises.

Eligibility: To be eligible, projects must be focused on improving the viability of off grid renewable
energy businesses and organizations, underserved individuals and communities in India without
access to grid connected power or with limited orintermittent accesslessthan 8 hours per day using
smallscale (under 1megawatt) clean energy systems.

Q
Source:https://www.insightsonindia.com/wp-content/uploads/2020/04/INSTA-PT-2020-Exclusive-Int
ernational-Relations.pdf

76 Consider the following statements.


1. Ionosphere is a layer abundant with ionized atoms and molecules touching both mesosphere and
exosphere.
2. Ionosphere grows and shrinks based on solar conditions or radiation.
3. NASAhas launched a satellite ICON to detect dynamic zones of Earth’s Ionosphere.

Select the correct answer using the codes below.


A. 1 and 2 only
B. 2 and 3 only
C. 1, 2 and 3
D. 3 only

Correct Answer : B

Answer Justification :

Justification: Ionosphereis an abundant layer of electrons and ionized atoms and molecules that
stretches from about 48 kilometers (30 miles) above the surface to the edge of space at about 965
km (600 mi), overlapping into the mesosphere and thermosphere.

This dynamic region grows and shrinks based on solar conditions and divides further into the sub-
regions: D, E and F; based on what wavelength of solar radiation is absorbed. The ionosphere is a
critical link in the chain of Sun-Earth interactions. This region is what makes radio communications
possible.

National Aeronautics and Space Administration (NASA) has launched a satellite ICON to detect
dynamic zones of Earth’s Ionosphere. The satellite Ionosphere Connection Explorer (ICON) was
launched from an aircraft over the Atlantic Ocean near the Florida coast.

66
Total Marks : 200
Online Prelims TEST - 31 (TEXTBOOK)
( InsightsIAS Mock Test Series for UPSC Preliminary Exam 2020 ) Mark Scored : 0

1. The ICON satellite will study the Earth’s Ionosphere. It includes various layers of the uppermost
atmosphere where free electrons flow freely.

2. The ICON mission is the 39th successful launch and satellite deployment by Pegasus rocket. This
mission is operated by the University of California.

3. It was originally planned to launch in late 2017 but delayed due to the problems with the Pegasus
XL rocket. It is equipped with 780-watt solar arrays to power the instruments.

Q Source:As mentioned above

77 Oslo Accords see in news concern which of these countries?

A. Egypt-Sudan
B. Israel-Palestine
C. Iraq-Iran
D. Turkmenistan-Afghanistan

Correct Answer : B

Answer Justification :

Justification: About the Oslo Accords: Formally known as the Declaration of Principles (DOP), the
pact established a timetable for the Middle East peace process. It planned for an interim Palestinian
government in Gaza and Jericho in the West Bank. Oslo II, officially called the Israeli-Palestinian
Interim Agreement on the West Bank and Gaza, expanded on Oslo I. It included provisions for the
complete withdrawal of Israeli troops from six West Bank cities and about 450 towns. Additionally,
the pact set a timetable for elections for the Palestinian Legislative Council.

Palestinians had threaten to quit Oslo Accords, if U.S. President Donald Trump announces his West
Asia peace plan.

Why? Palestinians’ concerns: The main worry is that this initiative will turn Israel’s “temporary
occupation (of Palestinian territory) into a permanent occupation”. The Palestinians see east
Jerusalem as the capital of their future state and believe Mr. Trump’s plan buries the two-state
solution that has been for decades the cornerstone of international West Asia diplomacy.

What’s the issue?

Under the Oslo Accords of the 1993, both Israel and the Palestinians agreed that the status of
settlements would be decided by negotiations. But the negotiations process has been all but dead
for several years now. Israel walked into East Jerusalem in 1967, and subsequently annexed it. For
Israel, Jerusalem is non-negotiable. The Palestinians want East Jerusalem as the capital of their
future state. Most of the world’s nations look at it as occupied territory

Q
Source:https://www.insightsonindia.com/wp-content/uploads/2020/04/INSTA-PT-2020-Exclusive-Int
ernational-Relations.pdf
67
Total Marks : 200
Online Prelims TEST - 31 (TEXTBOOK)
( InsightsIAS Mock Test Series for UPSC Preliminary Exam 2020 ) Mark Scored : 0

78 This sector’s share in Gross Value Added (GVA) to the economy has been nearly 50% averaging over
a period between 2009-2019:

A. Agriculture, Forestry and Fishing


B. Industry, including manufacturing
C. Financial, Real Estate & Professional Services
D. Services sector

Correct Answer : D

Answer Justification :

Justification: Please refer to the table below:

Q Source:Ch1: ES 19-20 V2

68
Total Marks : 200
Online Prelims TEST - 31 (TEXTBOOK)
( InsightsIAS Mock Test Series for UPSC Preliminary Exam 2020 ) Mark Scored : 0

79 The fundamental doctrine of the Yogacara school of Mahayana Buddhism is that


1. All phenomenal existence is fabricated by consciousness
2. Consciousness is the basis of all activities from birth to attaining enlightenment.

Select the correct answer using the codes below.


A. 1 only
B. 2 only
C. Both 1 and 2
D. None of the above

Correct Answer : C

Answer Justification :

Justification: There were two principal schools of Mahāyāna Buddhism in India – Yogācāra and
Madhyamaka, the followers of which are called Mādhyamikas..

The fundamental doctrine of the Yogacara school is "that all phenomenal existence is fabricated by
consciousness." Consciousness is the basis of all activities from birth to attaining enlightenment;
"...all is based upon the coming into being and the ceasing to be of consciousness, i.e., of
distinctions in the mind."

The name of the Madhyamakaschool, on the other hand, is a reference to the claim made of
Buddhism in general that it is a middle path (madhyamāpratipad) that avoids the two extremes of
eternalism—the doctrine that all things exist because of an eternal essence—and
annihilationism—the doctrine that things have essences while they exist but that these essences are
annihilated just when the things themselves go out of existence.

Q Source: Topics related to Buddhism

80 Regional Anti-Terrorist Structure (RATS) is associated with which of the following bodies?

A. G20
B. Shanghai Cooperation Organization (SCO)
C. Financial Action Task Force (FATF)
D. BRICS

Correct Answer : B

Answer Justification :

Justification: The Regional Anti-Terrorist Structure (RATS), headquartered in Tashkent,


Uzbekistan, is a permanent organ of the SCO which serves to promote cooperation of member
states against the three evils of terrorism, separatism and extremism.

It was established at the 2004 SCO summit, held in Tashkent, Uzbekistan. The Head of RATS is

69
Total Marks : 200
Online Prelims TEST - 31 (TEXTBOOK)
( InsightsIAS Mock Test Series for UPSC Preliminary Exam 2020 ) Mark Scored : 0

elected to a three-year term. Each member state also sends a permanent representative to RATS.

We have covered SCO in detail in previous tests.

Q Source:Based on previous test’s syllabus

81 Consider the following statements.


1. In a monarchy, the head of the state may not be an elected representative of the people.
2. The term ‘republic’ in the Constitution’s Preamble indicates that India has an elected head, the
President.

Select the correct answer using the codes below.


A. 1 only
B. 2 only
C. Both 1 and 2
D. None of the above

Correct Answer : C

Answer Justification :

Justification: A democratic polity can be classified into two categories—monarchy and republic.

In a monarchy, the head of the state (usually king or queen) enjoys a hereditary position, that
is, he comes into office through succession, e.g., Britain.
In a republic, on the other hand, the head of the state is always elected directly or indirectly
for a fixed period, eg, USA.
Therefore, the term ‘republic’ in our Preamble indicates that India has an elected head called
the president.
He is elected indirectly for a fixed period of five years. A republic also means two more things:
one, vesting of political sovereignty in the people and not in a single individual like a king;
second, the absence of any privileged class and hence all public offices being opened to every
citizen without any discrimination.

Q Source: Indian Polity: M Laxmikanth

82 The Allahabad inscription provides specific details of the military achievements and conquests of
which of the following rulers?

A. Allaudin Khilji
B. Samudragupta
C. Ranjit Singh
D. Harshvardhana

Correct Answer : B

70
Total Marks : 200
Online Prelims TEST - 31 (TEXTBOOK)
( InsightsIAS Mock Test Series for UPSC Preliminary Exam 2020 ) Mark Scored : 0

Answer Justification :

Learning: It provides specific details of Samudragupta’s military achievements and conquests from
which it is possible to delineate the extent of his empire. It also presents the Gupta Empire as the
centre of a complex network of political relationships.

Samudragupta’s initial military campaigns were directed towards areas that lay immediately beyond
his control.

For e.g. Line 14 of the inscription refers to Samudragupta capturing a king of the Kota family, while
Line 21 talks of him violently destroying several kings of Aryavarta and subordinating all the forest
kings.

The territories of the kings mentioned in these two lines seem to have been annexed, leading to an
extension of the Gupta empire over the Ganga-Yamuna valley up to Mathura and Padmavati
(Pawaya) in the west.

Q Source:Based on past year UPSC papers

83 Consider the following with reference to Unnat Bharat Abhiyan.


1. It is a flagship programme of the Ministry of Rural Development.
2. It aims to create a relationship between the society and the higher educational institutes.
3. It focuses primarily on the capacity building of public functionaries.

Select the correct answer using the codes below.


A. 1 only
B. 1, 2 and 3
C. 2 and 3 only
D. 2 only

Correct Answer : D

Answer Justification :

Justification: It is a flagship programme of the Ministry of Human Resources Development, with


the intention to enrich Rural India.

It aims to leverage the knowledge base and resources of the Premier Institutions of the country to
bring in transformational change in rural developmental process.

It also aims to create a vibrant relationship between the society and the higher educational
institutes, with the latter providing the knowledge and technology support to improve the
livelihoods in rural areas and to upgrade the capabilities of both the public and private
organisations in the society.

Under the Unnat Bharat Abhiyan 2.0, the institutions have been selected on a Challenge Mode and
the scheme has been extended to 750 reputed Higher Educational Institutes (both public and
private) of the country.
71
Total Marks : 200
Online Prelims TEST - 31 (TEXTBOOK)
( InsightsIAS Mock Test Series for UPSC Preliminary Exam 2020 ) Mark Scored : 0

IIT Delhi has been designated to function as the National Coordinating Institute for this programme
and the Ministry intends to extend the coverage to all the reputed Higher Educational Institutes, in
a phased manner.

Each selected institute would adopt a cluster of villages / panchayats and gradually expand the
outreach over a period of time.

Q Source: Major schemes

84 One of the world's richest fishing grounds is situated around the coast of Iceland. This is because
Iceland coasts provide
1. Coastal upwelling of nutrient rich colder water
2. Abundance of phytoplankton
3. Meeting ground of warm and cold currents
4. Shallow continental shelves in a cold climate

Consider the following with regards to Select the correct answer using the codes below.
A. 2 and 3 only
B. 1, 2, 3 and 4
C. 1 and 4 only
D. 3 only

Correct Answer : B

Answer Justification :

Justification: The rich fishing grounds around Iceland depend on highly fertile phytoplankton
communities.

Like plants on land, phytoplankton have the ability to use sunlight to make organic matter out of
non-organic

In order to flourish and grow, phytoplankton need nutrients such as nitrate, phosphate and silica.
Deep sea is normally richer in nutrients than surface sea, and the most fertile seas in the world are
found where the nutrient-rich deep sea water has easy access to the surface layers.

Icelandic seas are among the most fertile in the world. It has been estimated that phytoplankton
within Icelandic fishing limits fix about 120 million tons of carbon in organic material every year,
which approaches the amount in the Barent Sea, another area with fertile fishing grounds.

To the south and west of Iceland the nutrient-rich deep sea streams up by the continental shelf and
enriches the surface waters. At the sea current divide, southeast and northwest of the island the
warm sea from the south and the cold sea from the north meet and draw up nutrients from the
deep, throughout the whole summer. So the sea is also very fertile in those regions. Unstable
weather conditions around Iceland also contribute to the blending of deep sea water with surface
water which replenishes the nutrient content in the upper layers. This means that conditions for
phytoplankton, which is the base for animal life in the sea, are very good off the shores of Iceland.

72
Total Marks : 200
Online Prelims TEST - 31 (TEXTBOOK)
( InsightsIAS Mock Test Series for UPSC Preliminary Exam 2020 ) Mark Scored : 0

Q Source:Based on past year papers of UPSC

85 Korwa, Sawar and Hill Kharia are Particularly Vulnerable Tribal Groups (PVTGs) found in the state of

A. Assam
B. Bihar and Jharkhand
C. Madhya Pradesh and Odisha
D. Meghalaya

Correct Answer : B

Answer Justification :

Justification: Here are the list of PVTGs from two states – you can access the entire list at the Q
Source. It is good to scan through the list and try to remember if you have encountered them in
news sometime.

73
Total Marks : 200
Online Prelims TEST - 31 (TEXTBOOK)
( InsightsIAS Mock Test Series for UPSC Preliminary Exam 2020 ) Mark Scored : 0

Q Source:https://tribal.nic.in/DivisionsFiles/ListofPVTGs191212.pdf

86 AUMX, a maritime exercise, seen recently in news, was conducted for the first time between

A. India and ASEAN


B. Japan and Russia
C. India and Mongolia
D. ASEAN and USA

Correct Answer : D

Answer Justification :

Justification: The first ASEAN-US Maritime Exercise (AUMX) between regional bloc- Association
of Southeast Asian Nations (ASEAN) and United States was held at the Sattahip Naval Base in
Thailand.

The U.S. Navy and maritime forces from the Association of Southeast Asian Nations (ASEAN) will
begin the first ASEAN-U.S. Maritime Exercise (AUMX) with opening ceremonies at Sattahip Naval
Base, Thailand Sept. 2.

Co-led by the U.S. and Royal Thai navies, AUMX will consist of pre-sail activities in Thailand,
Singapore and Brunei, followed by a sea phase in international waters in Southeast Asia, including
the Gulf of Thailand and South China Sea. The exercise will conclude in Singapore.

In all, AUMX includes eight warships and four aircraft from seven countries, and more than a
thousand personnel representing all ten ASEAN member states and the United States.

AUMX promotes shared commitments to maritime partnerships, security and stability in Southeast
Asia. Participating nations include Brunei, Cambodia, Indonesia, Laos, Malaysia, Myanmar,
Philippines, Singapore, Thailand, United States and Vietnam.

Throughout the exercise, ASEAN member states and U.S. forces will operate together under a
combined task force structure, executing a variety of realistic scenarios designed to reinforce
interoperability in areas such as visit, board, search and seizure (VBSS), maritime domain
awareness, division tactics and maritime asset tracking.

Q Source:https://asean.usmission.gov/asean-u-s-maritime-exercise-begins-in-thailand/

87 The Welby Commission that was setup by the British Government to enquire into the unnecessary
expenditures by the government had which of these personalities as the Indian member?

A. Romesh Chandra Dutta


B. DadabhaiNaoroji
C. Mahadev Govind Ranade
D. Abdul Ghaffar Khan

74
Total Marks : 200
Online Prelims TEST - 31 (TEXTBOOK)
( InsightsIAS Mock Test Series for UPSC Preliminary Exam 2020 ) Mark Scored : 0

Correct Answer : B

Answer Justification :

Learning: DadabhaiNaoroji in his famous book Poverty and UnBritish Rule in India wrote his Drain
Theory.

He showed how India’s wealth was going away to England in the form of: (a) salaries,(b)savings, (c)
pensions, (d) payments to British troops in India and (e) profits of the British companies.

The British Government was forced to appoint the Welby Commission, with Dadabhai as the first
Indian as its member, to enquire into the Matter.

The Welby Commission's report, published in 1900, showed a number of cases where excessive or
unjust payments had been made by the Indian government.

Q Source:Based on past year UPSC papers

88 With reference to Indus Dolphins, consider the following statements.


1. They are amphibians.
2. They are a blind species that communicate through echo like bats do.
3. They are found only in India and Pakistan.

Select the correct answer using the codes below.


A. 1 and 2 only
B. 2 only
C. 2 and 3 only
D. 1 and 3 only

Correct Answer : C

Answer Justification :

Justification: Statement 1: They are mammals.

Mammals are warm-blooded, but amphibians are cold-blooded. Mammals have hairs on skin,
whereas amphibians have a bare and moistened skin. Mammals have mammary glands to feed the
young but amphibian newborns are not breast-fed

Statement 2 and 3: It is a blind species that communicates through echo like bats do, Indus
dolphins are one of the seven freshwater dolphins found across the world.

They are found only in India and Pakistan. In Punjab, they are confined to only a 185 km stretch
between Talwara and Harike Barrage in India’s Beas river in Punjab.

The most flourishing population of the Indus dolphin, platanistagangetica minor, is found across
Pakistan where their numbers are estimated to be around 1,800 over a stretch of 1,500 km of the

75
Total Marks : 200
Online Prelims TEST - 31 (TEXTBOOK)
( InsightsIAS Mock Test Series for UPSC Preliminary Exam 2020 ) Mark Scored : 0

Indus river.

Learning: It is listed by the IUCN as endangered on its red list of threatened species. IUCN
suspects the population size of the Indus river dolphins has reduced by more than 50% since 1944.

For the conservation of Indus dolphins – one of the world’s rarest mammals – the Punjab
government along with WWF-India are conducting the first organised census on their population.

Q Source:Based on past year UPSC papers

89 Consider the following with regards to Special Data Dissemination Standard (SDDS) on which a
report was published by IMF sometime back.
1. SDDS is a global benchmark for disseminating macroeconomic statistics to the public.
2. SDDS subscription is only available to macroeconomically stable economies as defined by IMF.

Select the correct answer using the codes below.


A. 1 only
B. 2 only
C. Both 1 and 2
D. None of the above

Correct Answer : A

Answer Justification :

Justification: The SDDS is a global benchmark for disseminating macroeconomic statistics to the
public. SDDS subscription indicates that a country meets the test of “good statistical citizenship.”
Countriesthatsubscribe to the SDDS agree to follow good practicesin four areas: the coverage,
periodicity, and timeliness of data; public access to those data; data integrity; and data quality.

According to the IMF’s “Annual Observance Report of the Special Data Dissemination Standard for
2018”, India failed to comply with multiple requirements prescribed in the Special Data
Dissemination Standard (SDDS).

The report lists three types of deviations from SDDS: The first deals with delays in data
dissemination from the periodicity prescribed in the SDDS. The second occurs when member
countries do not list a data category in their Advance Release Calendars (ARC) despite the category
being mandated by the SDDS. The third deviation occurs when data is not disseminated at all for a
particular period.

Q
Source:https://www.insightsonindia.com/wp-content/uploads/2020/04/INSTA-PT-2020-Exclusive-Int
ernational-Relations.pdf

90 Consider the following with regards to banking in India.


1. Bank nationalizationprogramme was undertaken only after the 1991 economic reforms in India.
2. Public Sector Banks (PSBs) account for a larger market share in banking and credit allocation
76
Total Marks : 200
Online Prelims TEST - 31 (TEXTBOOK)
( InsightsIAS Mock Test Series for UPSC Preliminary Exam 2020 ) Mark Scored : 0

compared to private sector banks.

Select the correct answer using the codes below.


A. 1 only
B. 2 only
C. Both 1 and 2
D. None of the above

Correct Answer : B

Answer Justification :

Justification: In 2019, India completed the 50thanniversary of the banknationalizationprogramme


undertaken in 1969. It is, therefore,apt to celebrate theaccomplishments of the389,956 officers,
295,380 clerks, and 121,647sub-staff who work in public sector banks(PSBs). As PSBs account for
70 per cent ofthe market share in banking, an assessmentof the state of India’s public sector
banks(PSBs) is apposite.

For this reason, the onus ofsupporting the Indian economy and fostering its economic development
falls on PSBs.

Yet, on every performance parameter, PSBs are inefficient compared to their peer groups.In 2019,
every rupee of taxpayer money invested in PSBs, on average, lost 23 paise. Incontrast, every rupee
of investor money invested NPBs on average gained 9.6 paise.

Also, credit growth in PSBs has been much lower than NPBs for the last several years. The survey
suggests solutions that can make PSBs more efficient so that they are able toadeptly support the
nation in its march towards being a $5 trillion economy.

Q Source:Ch on Nationalization: ES19-20

91 With reference to the District Development coordination and Monitoring committee (DISHA),
consider the following statements.
1. The committee monitors the implementation of government programmes in the district to bring
about greater synergy between various stakeholders.
2. It is chaired by the District Collector/District Magistrate or Deputy Commissioner.
3. It looks into alleged irregularities received in respect of theimplementation of the programmes and
closely reviews the flow of funds for the government schemes.

Select the correct answer using the codes below.


A. 1 and 2 only
B. 1 and 3 only
C. 2 and 3 only
D. 1, 2 and 3

Correct Answer : B

77
Total Marks : 200
Online Prelims TEST - 31 (TEXTBOOK)
( InsightsIAS Mock Test Series for UPSC Preliminary Exam 2020 ) Mark Scored : 0

Answer Justification :

Background: A large number of significant and ambitious programmes are being implemented
forimprovement of infrastructure and for improving human development and well-being of the
people.

District Development coordination and Monitoring committee (DISHA) was formed with a view to
fulfill the objective of ensuring a better coordination among all the electedrepresentatives in
Parliament, State Legislatures and Local Governments (Panchayati Raj Institutions/Municipal
Bodies) for efficient and time-bound development of districts in our country.

Justification: Composition:

The Chairperson of the DISHA should be a Member of Parliament (Lok Sabha) elected from the
district, nominated by the Ministry of Rural Development. The other MPs may act as co-chairperson.

The Member Secretary of the DISHA should be the District collector /District Magistrate/ Deputy
Commissioner except in cases where specific exemption has been given by the Union Government.

You can find a list of other members in the Q Source link. At least one of the above persons shall be
a woman and; At least one of theabove persons shall be from either a scheduled caste or a
scheduled tribe category.

Terms of Reference

Ensure that all programmes are implemented in accordance with the programme Guidelines.
Facilitate coordinated solution to remove constraints of any kind
Facilitate smooth implementation of priorities determined by the DPC
Resolve matters related to provision of land and space for faster roll out of priorities.
Guide DPCs about all the national programmes and how they can be leveraged for
transformation of the district
Identify issues for follow up in Parliament, state Assemblies and Local Governments for timely
achievement of objectives.
intensively monitor all time bound national initiatives for universal coverage
Recommend improvements in design of approved programmes and suggest mid-course
corrections to address implementation constraints.
Look into complaints/alleged irregularities received in respect of the implementation of the
programmes
Closely review the flow of funds including the funds allocated, funds released by both Centre
and the State, utilization and unspent balances under each Scheme

Q Source:http://rural.nic.in/sites/default/files/DISHA_Guidelines.pdf

http://rural.nic.in/disha

92 SARAS Fairs are being organized by the government to promote

A. Defence Technology
B. Wildlife Conservation in Wetlands

78
Total Marks : 200
Online Prelims TEST - 31 (TEXTBOOK)
( InsightsIAS Mock Test Series for UPSC Preliminary Exam 2020 ) Mark Scored : 0

C. SHG made products


D. Philology

Correct Answer : C

Answer Justification :

Background:Marketing of products of the SHG women is one of the major concerns of the Ministry
of Rural Development.

In order to empower the rural women artisans and to bring them above the poverty line, through
access to better market and marketing systems, the Ministry of Rural Development has been
supporting the organisation of exhibitions under the brand name of SARAS where Self Help Groups
from different states participate and sell their products.

At present, two SARAS Fairs in a State are sponsored by the Ministry during a financial year by way
of providing assistance uptoRs 40.00 lakh per fair in Metropolitan cities (Bangalore, Mumbai,
Chennai, Delhi, Kolkata and Hyderabad) and uptoRs 35.00 lakhs per fair in non-Metropolitan cities.

In addition, the Ministry has been organizing SARAS fairs at Delhi as a part of India International
Trade Fair (IITF) and also as ShishirMela in DilliHaat.

The SARAS AajeevikaMela 2018 in New Delhi is an endeavour of DAY-NRLM to provide to the rural
women producers a national platform and an opportunity to showcase their products, and seek
buyers, either individuals or bulk buyers for sale nationally / internationally.

On this occasion, the Ministry will also be bringing out a directory of the products of all the women
who will be showcasing their products in the AajeevikaMela 2018.

Q Source: http://rural.nic.in/

93 Adilabad Dokrais an ancient metal craft popular in the tribal regions of

A. Telangana
B. Madhya Pradesh
C. Chhattisgarh
D. Kerala

Correct Answer : A

Answer Justification :

Justification: The uniqueness of this art form is that no two sculptures are the same, as they are
not made out of a pre-designed cast.

The craftsmen of this art form belong to the Woj community, commonly referred to as Wojaris

79
Total Marks : 200
Online Prelims TEST - 31 (TEXTBOOK)
( InsightsIAS Mock Test Series for UPSC Preliminary Exam 2020 ) Mark Scored : 0

or Ohjas in the rural areas of Telangana.


They use brass as their main material and an ancient casting technique called ‘cire perdue’.
As part of this technique, the craftsmen use clay and design a model of the sculpture that they
want to create. They wrap the clay with wax threads before baking the mould, so that the wax
melts away and the molten metal is poured into the mould.
The common items made by the craftsmen include small idols and statues of tribal deities,
jewellery, bells, small-scale animal sculptures and others.

Q Source: From the GI list – Tribal products

94 With reference to the Sustainable Urban Transport Project (SUTP), consider the following
statements.
1. SUTP is funded by the UNDP, GEF, World Bank, Government of India and participating cities and
states.
2. The Ministry of Road Transport and Highwaysis the nodal agency for the implementation of the
project.

Select the correct answer using the codes below.


A. 1 only
B. 2 only
C. Both 1 and 2
D. None

Correct Answer : A

Answer Justification :

Justification: Achieving a sustainable urban transport system has become a primary objective with
the adoption of National Urban Transport Policy (NUTP) by the GoI in 2006.

The Ministry of Housing and Urban Affairshas initiated the SUTP with support of GEF and the
World Bank to foster a long-term partnership between GoI and state/local governments in the
implementation of a greener environment under the ambit of the NUTP. The aim of the project is to
achieve a paradigm shift in India’s urban transport systems in favour of sustainable development.

The MoHUAis the nodal agency for the implementation of the project.

The Project Development Objectives are

to promote environmentally sustainable urban transport nationally;


to improve the usage of environment-friendly transport modes through demonstration
projects in selected cities

Learning: World Bank funded projects require performance monitoring indicators. The SUTP key
overall indicators to measure performance in achieving the project development and global
environmental objective are:

80
Total Marks : 200
Online Prelims TEST - 31 (TEXTBOOK)
( InsightsIAS Mock Test Series for UPSC Preliminary Exam 2020 ) Mark Scored : 0

The number of cities that develop an identifiable urban transport planning process (i.e.,
managed by professional units of government, following certain procedures and guidance, and
involving various level of analytical work) increases.
Mode shares in Hubli-Dharwad, Pimpri-Chinchwad, and Naya Raipur become more
sustainable by project end.

A significant amount of co-benefits are achieved as forecast transport CO2 emissions in the
demonstration cities are lower than their “business-as-usual” or “without-project” forecasts.

Q Source: http://mohua.gov.in/cms/SUTP.php

95 Government is implementing the National Child Labour Project (NCLP) Scheme for rehabilitation of
child labour including domestic helps. Consider the following about it.
1. Surveys are conducted on a regular basis by the District Project Societies headed by District
Magistrate to identify child labour.
2. Children in the age group of 9-14 years rescued from work are enrolled in the NCLP Special
Training Centres.
3. Children rescued in the age group of 5-8 years are directly linked to the formal education system
through a close coordination with the Sarva Shiksha Abhiyan (SSA).

Select the correct answer using the codes below.


A. 1 only
B. 1 and 2 only
C. 1, 2 and 3
D. 3 only

Correct Answer : C

Answer Justification :

Justification: Under this Scheme, survey is conducted on regular basis by the District Project
Societies headed by District Magistrate/Collector to identify child labour including children working
as domestic helps.

Children in the age group of 9-14 years, rescued/withdrawn from work are enrolled in the NCLP

81
Total Marks : 200
Online Prelims TEST - 31 (TEXTBOOK)
( InsightsIAS Mock Test Series for UPSC Preliminary Exam 2020 ) Mark Scored : 0

Special Training Centres, where they are provided with bridge education, vocational training, mid
day meal, stipend, health care, etc. before being mainstreamed into formal education system.

Children in the age group of 5-8 years are directly linked to the formal education system through a
close coordination with the Sarva Shiksha Abhiyan (SSA). The Scheme also envisages awareness
activities against the evil of child labour and enforcement of Child & Adolescent Labour (Prohibition
& Regulation) Act, 1986.

Consequent upon strengthening the legislative framework for complete prohibition of employment
of children below 14 years through amendment in the Child Labour (Prohibition & Regulation) Act,
1986, Government has provided robust enforcement mechanism in the form of an online portal
PENCIL (Platform for Effective Enforcement of No Child Labour).

Q Source: Major schemes

96 Consider the following statements about the International Centre For Alternative Dispute Resolution
(ICADR), India.
1. The ICADR is an autonomous organization.
2. The Regional Centres of ICADR are fully funded and supported by the respective State Governments
in India.
3. Union Minister of Law and Justice is the Chairperson of ICADR.

Select the correct answer using the codes below.


A. 1 and 3 only
B. 1 and 2 only
C. 2 and 3 only
D. 1, 2 and 3

Correct Answer : B

Answer Justification :

Justification: The Government of India thought it necessary to provide a new forum and procedure
for resolving international and domestic commercial disputes quickly through ADR.

The ICADR is an autonomous organization with its headquarters at New Delhi and Regional Centres
at Hyderabad and Bengaluru. The Regional Centres of ICADR are fully funded and supported by the
respective State Governments.

Dr. H.R. Bhardwaj (Founder Chairperson), Former Governor of Karnataka is the Patron of ICADR.
At the regional level, the Chief Justice of the concerned High Court is the Patron of the Regional
Centre of ICADR. Hon’ble Chief Justice of India is the Chairperson of ICADR. The Governing Council
of ICADR comprises of several eminent personalities drawn from various fields.

To facilitate the conduct of arbitral proceedings that the parties have agreed to conduct under the
Arbitration rules, the ICADR will-

82
Total Marks : 200
Online Prelims TEST - 31 (TEXTBOOK)
( InsightsIAS Mock Test Series for UPSC Preliminary Exam 2020 ) Mark Scored : 0

(a) perform the functions of the appointing authority whenever-

the ICADR has been so designated by the parties in the arbitration clause of their contract or
in a separate agreement, or
the parties have agreed to submit a dispute to the ICADR under the Arbitration rules without
specifically designating it as the appointing authority; and

(b) provide the administrative services herein specified when required by the agreement, or when
requested by all parties, or by the arbitral tribunal with the consent of the parties.

Q Source: http://icadr.nic.in/#

http://legalaffairs.gov.in/arbitration

97 Consider the following with regards to disinvestment in India.


1. The present policy discourages buy-back of shares of smaller PSUs by large PSUs having huge
surplus.
2. NITI Aayog has been mandated to identify PSUs for strategic disinvestment.
3. Disinvesting through creation of exchange traded funds (ETFs) of PSU shares is a part of the
disinvestment strategy of the government.

Select the correct answer using the codes below.


A. 1 and 2 only
B. 2 and 3 only
C. 3 only
D. 1, 2 and 3

Correct Answer : B

Answer Justification :

Justification: Departmentof Investment and Public Asset Management (DIPAM) has laid down
comprehensive guidelines on“Capital Restructuring of CPSEs” in May, 2016 by addressing various
aspects, such as, payment ofdividend, buyback of shares, issues of bonus shares and splitting of
shares. The Government has beenfollowing an active policy on disinvestment in CPSEs through the
various modes:

i. Disinvestment through minority stake sale in listed CPSEs to achieve minimum public
shareholding norms of 25 per cent. While pursuing disinvestment of CPSEs, the Government
will retain majority shareholding, i.e., at least 51 per cent and management control of the
Public Sector Undertakings;

ii. Listing of CPSEs to facilitate people’s ownership and improve the efficiency of companies
through accountability to its stake holders - As many as 57 PSUs are now listed with total
market capitalisation of over ` 13 lakh crore.

iii. Strategic Disinvestment;

83
Total Marks : 200
Online Prelims TEST - 31 (TEXTBOOK)
( InsightsIAS Mock Test Series for UPSC Preliminary Exam 2020 ) Mark Scored : 0

iv. Buy-back of shares by large PSUs having huge surplus;

v. Merger and acquisitions among PSUs in the same sector;

vi. Launch of exchange traded funds (ETFs) - an equity instrument that tracks a particular
index. The CPSE ETF is made up of equity investments in India’s major public sectorcompanies like
ONGC, REC, Coal India, Container Corp, Oil India, Power Finance, GAIL,BEL, EIL, Indian Oil and
NTPC; and

vii. Monetization of select assets of CPSEs to improve their balance sheet/reduce their debts andto
meet part of their capital expenditure requirements.

NITI Aayog has been mandated to identify PSUs for strategic disinvestment. For thispurpose,
NITIAayog has classified PSUs into “high priority” and “low priority”, based on (a) National Security
(b)Sovereign functions at arm’s length, and (c) Market Imperfections and Public Purpose. The
PSUsfalling under “low priority” are covered for strategic disinvestment. To facilitate quick
decisionmaking, powers to decide the following have been delegated to an Alternative Mechanism
in all thecases of Strategic Disinvestment of CPSEs where Cabinet Committee on Economic Affairs
(CCEA)has given ‘in principle’ approval for strategic disinvestment.

Q Source:Ch 9: ES 19-20

98 Which of the following Indian cities are listed in the UNESCO Creative Cities Network (UCCN)?
1. Jaipur
2. Varanasi
3. Bengaluru
4. Chandigarh

Select the correct answer using the codes below.


A. 1, 2, 3 and 4
B. 1 and 2 only
C. 3 and 4 only
D. 1, 2 and 4 only

Correct Answer : B

Answer Justification :

Justification: As of November, 2019, there are five Indian cities in UNESCO Creative Cities
Network (UCCN) as follows: 1. Jaipur-Crafts and Folk Arts (2015). 2. Varanasi-Creative city of Music
(2015). 3. Chennai-Creative city of Music (2017). 4. Mumbai – Film (2019). 5. Hyderabad –
Gastronomy (2019).

The UNESCO Creative Cities Network (UCCN) was created in 2004 to promote cooperation with
and among cities that have identified creativity as a strategic factor for sustainable urban
development. Objective: placing creativity and cultural industries at the heart of their development
plans at the local level and cooperating actively at the international level.

84
Total Marks : 200
Online Prelims TEST - 31 (TEXTBOOK)
( InsightsIAS Mock Test Series for UPSC Preliminary Exam 2020 ) Mark Scored : 0

The Network covers seven creative fields: Crafts and Folk Arts, Media Arts, Film, Design,
Gastronomy, Literature and Music. Joining the Network, cities commit to sharing their best
practices and developing partnerships involving the public and private sectors as well as civil
society in order to:

Strengthen the creation, production, distribution and dissemination of cultural activities,


goods and services;
Develop hubs of creativity and innovation and broaden opportunities for creators and
professionals in the cultural sector;
Improve access to and participation in cultural life, in particular for marginalized or
vulnerable groups and individuals;
Fully integrate culture and creativity into sustainable development plans.

Q Source:Insights current affairs

99 The criteria for declaring any area as a “Scheduled Area” under the Fifth Schedule has been defined
in/by the

A. Constitution of India
B. Protection of Civil Rights Act
C. Executive orders and conventions
D. Supreme Court of India

Correct Answer : C

Answer Justification :

Justification: In this Constitution, the expression “Scheduled Areas” means such areas as the
President may by order declare to be Scheduled Areas.

The criteria for declaring any area as a “Scheduled Area” under the Fifth Schedule are:

Preponderance of tribal population,


Compactness and reasonable size of the area,
A viable administrative entity such as a district, block or taluk, and
Economic backwardness of the area as compared to the neighbouring areas.

These criteria are not spelt out in the Constitution of India but have become well established.
Accordingly, since the year 1950 to 2007 Constitutional Order relating to Scheduled Areas have
been notified.

Q Source:https://tribal.nic.in/declarationof5thSchedule.aspx

100 Peste des Petits Ruminants (PPR)is a highly contagious animal disease mainly affecting

A. Tigers
B. Sheep and goat
85
Total Marks : 200
Online Prelims TEST - 31 (TEXTBOOK)
( InsightsIAS Mock Test Series for UPSC Preliminary Exam 2020 ) Mark Scored : 0

C. Bats
D. Cows

Correct Answer : B

Answer Justification :

Justification: Peste des Petits Ruminants (PPR), also known as sheep and goat plague, is a highly
contagious animal disease affecting small ruminants. Once introduced, the virus can infect up to 90
percent of an animal heard, and the disease kills anywhere from 30 to 70 percent of infected
animals. The PPR virus does not infect humans.

PPR was first described in 1942 in Côte d'Ivoire. Since then the disease has spread to large regions
in Africa, the Middle East and Asia. Today, more than 70 countries have confirmed PPR within their
borders, and many countries are at risk of the disease being introduced. These regions are home to
approximately 1.7 billion heads – roughly 80 percent – of the global population of sheep and goats.

Q Source:www.fao.org/ppr/en

86

You might also like